TMC Practice Exam B, TMC Practice Questions A

¡Supera tus tareas y exámenes ahora con Quizwiz!

A patient recently underwent a thoracotomy and is now receiving IPPB QID. The patient's inspiratory capacity has diminished over the past several days and the patient is becoming increasingly short of breath. Chest radiograph shows left lower lobe radiodensity with a concave upper border. The respiratory therapist would conclude that the patient has developed A. a pleural effusion. B. atelectasis. C. a pneumothorax. D. pneumonia.

A. a pleural effusion.

A patient involved in a motor vehicle accident has sustained a long bone fracture and remains in traction. The patient suddenly complains of chest pain and develops tachypnea and tachycardia. The respiratory therapist should A. administer 100% oxygen. B. recommend heparin. C. recommend streptokinase. D. request a chest X-ray.

A. administer 100% oxygen

A 55 year old patient admitted to the emergency department has a history of hypertension. The patient is conscious, diaphoretic and complaining of chest pain. The respiratory therapist's first response should be to A. administer 100% oxygen. B. place the patient on a cardiac monitor. C. administer nitroglycerin sublingual. D. insert a peripheral intravenous line.

A. administer 100% oxygen.

A patient's cardiac output is increased and his QS/QT is calculated to be 20%. Based upon this information, the respiratory therapist would inform the physician this patient has A. an elevated shunt. B. a reduced cardiac index. C. increased pulmonary vascular resistance. D. normal lung mechanics.

A. an elevated shunt.

A patient with a history of cardiovascular disease has been brought to the emergency department. He is intubated and is being manually ventilated. The ECG monitor shows a sinus rhythm with a rate of 50 beats/min. The patient's pulse is weak and irregular. The nurse has been unable to place a central or peripheral venous catheter. The therapist should recommend endotracheal instillation of A. atropine. B. lidocaine. C. epinephrine. D. naloxone.

A. atropine.

A patient admitted to the ED is noted to have pulse and blood pressure variations with respirations. This is most indicative of A. cardiac tamponade. B. severe pneumonia. C. congestive heart failure. D. large pleural effusion.

A. cardiac tamponade.

A patient has 2 mediastinal chest tubes following a thoracotomy. The respiratory therapist notes that there is continuous bubbling in the water seal chamber of the chest drainage system. The therapist should A. check for a leak in the system. B. add water to the water seal chamber. C. increase the amount of suction. D. advance the chest tube 2 cm.

A. check for a leak in the system.

A respiratory therapist enters a patient's room during oxygen rounds. The patient has end-stage emphysema and appears to be sleeping. The patient doesn't respond to questions and his pulse is 20 bpm. The therapist should immediately A. confirm DNR status. B. go get help. C. begin rescue ventilation. D. begin chest compressions.

A. confirm DNR status.

During a cardiopulmonary stress test on a 55-year-old man, the respiratory therapist notes the following changes: Heart rate increases Blood pressure increases VD/VT ratio decreases Respiratory rate increases The therapist should A. continue the test. B. terminate the test. C. contact the physician. D. administer amiodarone.

A. continue the test.

During inline suctioning of a patient's endotracheal tube, 100% oxygen is being supplied via the ventilator. The respiratory therapist observes several premature ventricular contractions on the ECG monitor. The therapist should A. decrease the suctioning time per pass. B. use a smaller suction catheter. C. increase oxygenation time. D. decrease the suction pressure.

A. decrease the suctioning time per pass.

The physician requests assistance in establishing a patent airway for a conscious patient who requires frequent suctioning. The respiratory therapist should recommend insertion of a/an A. nasopharyngeal airway. B. endobronchial airway. C. oropharyngeal airway. D. nasal oxygen catheter.

A. nasopharyngeal airway.

While measuring peak flow on a patient with well-controlled asthma, the respiratory therapist notes that the peak flow meter consistently reads 200 L/min despite excellent patient effort. The most likely explanation for these results is that the A. peak flow meter is clogged. B. patient's asthma has worsened. C. patient is fatigued. D. peak flow meter is accurate.

A. peak flow meter is clogged.

The most probable cause of air bronchograms and increased density on a chest x-ray is A. pneumonia. B. pulmonary edema. C. pulmonary embolism. D. pleural effusion.

A. pneumonia.

As the respiratory therapist assesses a patient who is in a semi-fowler position, he/she finds that the patient's jugular vein extends approximately 7 cm above his sternal angle. The therapist should recommend initiation of which therapy? A. positive inotrope B. antibiotic C. long-acting Beta-agonist D. hyperinflation

A. positive inotrope

The high pressure alarm is sounding frequently on the ventilator of a 58 year-old patient who is being mechanically ventilated following major orthopedic surgery. The set rate on the ventilator is 12/min and auscultation reveals bilateral vesicular breath sounds. Current vital signs include: heart rate 130/min, respiratory rate 36/min, blood pressure 125/88 mmHg, temperature 37.1°C (98.8°F), and SpO2 96%. The patient appears quite agitated. The patient's respiratory pattern is most likely the result of A. post-operative pain. B. a viral infection. C. hypoxemia. D. lactic acidosis.

A. post-operative pain.

A 70-kg (154 lb) patient with emphysema is receiving mechanical ventilation. Current ventilator settings are as follows: VC, SIMV; VT 550 mL, respiratory rate 12/min, FIO2 0.30. The patient is awake and alert and does not appear to be in any distress. The total respiratory rate is 14/min. ABG results are as follows: pH 7.35, PaCO2 58 torr; PaO2 65 torr; HCO3 30 mEq/L. His SpO2 is 94% and MIP is -30 cm H2O. This patient is most likely A. ready for a spontaneous breathing trial. B. in an acute exacerbation. C. oversedated. D. in need of PEEP therapy.

A. ready for a spontaneous breathing trial.

All of the following are goals of bronchial hygiene therapy EXCEPT A. reverse the underlying disease process. B. improve mobilization of retained secretions. C. improve pulmonary gas exchange. D. reduce the work of breathing.

A. reverse the underlying disease process.

The respiratory therapist receives an order to administer bronchodilator therapy with albuterol. To reduce the chance of cross contamination, the therapist should A. select a metered dose inhaler. B. utilize a dry powder inhaler. C. replace the small volume nebulizer every 24 hours. D. wipe the surface of the nebulizer with alcohol every evening.

A. select a metered dose inhaler.

While administering an intermittent positive pressure breathing treatment to a post-operative patient with atelectasis, the respiratory therapist determines that the ventilator is self-cycling between the patient-initiated breaths. The therapist should adjust the A. sensitivity. B. expiratory time. C. flow. D. inspiratory pressure.

A. sensitivity.

A 42 year-old trauma patient in the ED has been intubated with a 6.5 mm oral endotracheal tube equipped with a high-residual-volume, low-pressure cuff. The respiratory therapist notes that a cuff pressure of 42 cm H2O is necessary to achieve a minimal occluding volume. This would indicate that the A. tube is not of the appropriate size. B. pilot balloon and line are obstructed. C. pressure manometer is defective. D. cuff has herniated over the tip of the tube.

A. tube is not of the appropriate size.

A patient who suffered trauma in an ATV accident is being monitored in the ICU. A pulmonary artery catheter has been placed and the following data is available: PvO2 46 torr PCWP 19 mm Hg PAP (mean) 10 mm Hg CVP 12 cm H2O Cardiac Output 3L/min The respiratory therapist should recommend 1. IV fluid challenge 2. positive inotropic agent 3. inhaled nitric oxide 4. diuretic therapy A. 1 and 3 B. 2 and 4 C. 3 and 4 D. 1 and 2

B. 2 and 4

What is the normal VD/VT ratio for a patient breathing room air? A. 5 - 15% B. 20 - 40% C. 45 - 55% D. 65 - 75%

B. 20 - 40%

A patient is being ventilated in the intensive care unit. The following data is obtained: Mode VC, SIMV Mandatory rate 12 b/min Total rate 18 b/min VT 800 mL FIO2 0.60 PIP 31 cm H2O PEEP 10 cm H2O pH 7.41 PaCO2 40 torr PaO2 95 torr SaO2 96% HCO3- 23 mEq/L BE +1 mEq/L PECO2 30 torr Hb 15 g/dL What should the respiratory therapist report as the VD/VT ratio? A. 15% B. 25% C. 40% D. 50%

B. 25%

The ability to distinguish central apnea from obstructive apnea during a sleep study requires the respiratory therapist to monitor 1. electrocardiogram. 2. electroencephalogram. 3. nasal air flow. 4. chest wall impedance. A. 1 and 2 only B. 3 and 4 only C. 2, 3, and 4 only D. 1, 2, and 3 only

B. 3 and 4 only

Transcutaneous monitoring of PO2 values will correlate well with arterial blood gas PO2 values in which of the following situations? 1. Hypotension 2. Hypothermia 3. Pneumonia A. 1 only B. 3 only C. 1 and 2 only D. 2 and 3 only

B. 3 only

A hyperresonant percussion note is associated with which of the following pathologies? 1. Hemothorax 2. Pleural effusion 3. Tension Pneumothorax 4. Chronic bronchitis A. 1, 3 B. 3, 4 C. 1, 2 D. 2, 4

B. 3, 4

In order to verify the accuracy of a lab-based spirometer device, the respiratory therapist should utilize a A. rotameter. B. 3.0 L syringe. C. Wright respirometer. D. pneumotachometer.

B. 3.0 L syringe.

The respiratory therapist is reviewing a patient's chart prior to performing a bedside assessment to determine readiness for weaning from ventilatory assistance. The patient weighs 60 kg (132 lb). The therapist would expect the patient to be able to achieve a spontaneous tidal volume of A. 200 - 355 mL B. 300 - 480 mL C. 400 - 620 mL D. 600 - 720 mL

B. 300 - 480 mL

What value for the apnea-hypopnea index (AHI) is consistent with mild obstructive sleep apnea? A. Less than 5 B. 5 to 15 C. 16 to 30 D. Greater than 30

B. 5 to 15

A patient on the general medical ward is on a 28% air entrainment mask with the flowmeter set at 5 L/min. What is the total flow delivered to the patient? A. 5 L/min B. 55 L/min C. 88 L/min D. 140 L/min

B. 55 L/min

A 30-year-old male who weighs 68 kg (150 lb) has a minute ventilation of 9 L/min. and a respiratory rate of 20/min. What is his alveolar minute volume? A. 4 L/min. B. 6 L/min. C. 8 L/min. D. 12 L/min.

B. 6 L/min.

The respiratory therapist obtains a SpO2 reading of 90% on a patient receiving oxygen therapy via 50% venti-mask. This would indicate a PO2 value of approximately A. 55 torr B. 60 torr C. 65 torr D. 70 torr

B. 60 torr

Which of the following patients would most likely benefit from pressure support ventilation? A. An intubated patient with an absent respiratory drive. B. A patient on SIMV with a set rate of 12/min and total rate of 24/min. C. A patient with acute lung injury. D. A patient who requires short-term post-operative ventilatory support.

B. A patient on SIMV with a set rate of 12/min and total rate of 24/min.

A 60 kg (132 lb) patient is being mechanically ventilated with the following settings: VC, A/C; VT 500 mL, respiratory rate 12/min, FIO2 1.00 and 10 cm H2O PEEP. The patient's peak airway pressure is 60 cm H2O and his SpO2 is 85%. A current chest x-ray shows diffuse bilateral infiltrates. Which of the following is the most appropriate action in order to reduce peak airway pressure? A. Increase the frequency. B. Change to airway pressure release ventilation. C. Decrease the inspiratory time. D. Increase PEEP to 15 cm H2O.

B. Change to airway pressure release ventilation.

A 64-year-old, 70 kg (154 lb) man with severe COPD is receiving independent (differential) lung ventilation following thoracotomy and right lower lobectomy. Which of the following setting combinations would be most appropriate for this patient? A. Right lung 50 mL; left lung 650 mL B. Right lung 150 mL; left lung 550 mL C. Right lung 350 mL; left lung 350 mL D. Right lung 550 mL; left lung 150 mL

B. Right lung 150 mL; left lung 550 mL

While examining the chest drainage system of a mechanically-ventilated patient following thoracotomy, the respiratory therapist observes bubbling in the water-seal chamber during inspiration. This would indicate A. a leak in the chest drainage system. B. air leaving the pleural space. C. excessive pressure from the suction regulator. D. inadequate water level in the water-seal chamber.

B. air leaving the pleural space.

During mechanical ventilation, a patient initiates the breath and then the ventilator controls the remaining variables for that breath. This describes a/an A. spontaneous breath. B. assisted breath. C. mandatory breath. D. supported breath.

B. assisted breath.

A 59 year-old post CABG patient has a C(a-v)O2 that has increased from 5 mL/dL to 8 mL/dL. The respiratory therapist should report to the physician that the patient's A. hemoglobin is increasing. B. cardiac output is decreasing. C. VD/VT ratio has increased. D. oxygen consumption has decreased.

B. cardiac output is decreasing.

During bedside monitoring the respiratory therapist notices a dampened waveform on the arterial line graphic. To restore the graphic to normal, the therapist should first A. verify the position of the transducer. B. check the transducer dome for air bubbles. C. flush the catheter with heparin solution. D. attempt to draw blood from the arterial line.

B. check the transducer dome for air bubbles.

After a patient undergoes a thoracentesis, the respiratory therapist notes that the obtained pleural fluid is clear with a slight straw color. This fluid is most likely the result of A. empyema. B. congestive heart failure. C. lung carcinoma. D. hemothorax.

B. congestive heart failure.

A 58 year old male patient is hypoxic on volume control ventilation and is receiving 15 cm H2O of PEEP on an FIO2 0.60. Shortly after increasing the PEEP therapy to 18 cm H2O, the respiratory therapist notes that the systemic blood pressure has fallen from 110/72 mm Hg to 94/50 mm Hg and the cardiac output has fallen from 4.3 L/min. to 2.5 L/min. The most appropriate action at this time would be to A. maintain the present therapy and re-evaluate the patient in thirty minutes. B. decrease the PEEP to 15 cm H2O and increase the FIO2 to 0.70. C. decrease the PEEP to 12 cm H2O and maintain the FIO2 at 0.60. D. discontinue the PEEP and increase the FIO2 to 0.80.

B. decrease the PEEP to 15 cm H2O and increase the FIO2 to 0.70.

An optimal PEEP study is initiated on a patient receiving mechanical ventilation. The respiratory therapist first places the patient on a PEEP of 10 cm H2O for 20 minutes with no adverse effects. The PEEP is increased to 15 cm H2O and the patient's heart rate rises significantly with a severe fall in the blood pressure. Based upon the above information, the therapist should conclude that the patient is suffering from A. peripheral vasoconstriction. B. hypovolemia. C. increased venous return. D. increased SVR

B. hypovolemia.

The respiratory therapist is called to ICU to evaluate a patient on continuous flow CPAP (8 cm H2O, FIO2 0.30) who is showing signs of respiratory distress. The patient's SpO2 has changed from 94% to 90%. The therapist observes that the CPAP pressure manometer displays negative pressure during inspiration. The therapist should A. place the patient back on the ventilator at the previous settings. B. increase inspiratory flow to the CPAP system. C. assess the patient for excess secretions. D. increase the CPAP to 10 cm H2O.

B. increase inspiratory flow to the CPAP system.

A patient is receiving IPPB with a Bird Mark-7. The respiratory therapist notes that the patient is generating negative pressure at the beginning of inspiration although the machine does not switch into the inspiratory phase. The respiratory therapist should A. decrease the flow. B. increase the sensitivity. C. adjust the apnea timer. D. decrease the pressure.

B. increase the sensitivity.

While instructing a patient prior to a vital capacity maneuver, the respiratory therapist should direct the patient to A. exhale to residual volume and inhale to inspiratory capacity. B. inhale to total lung capacity then exhale to residual volume. C. exhale normally then inhale to total lung capacity. D. inhale normally then exhale to functional residual capacity.

B. inhale to total lung capacity then exhale to residual volume.

Heliox therapy is indicated for treatment of patients with post extubation stridor because of the A. ability to effectively deliver humidity. B. low density of the gas mixture. C. high molecular weight of the gas mixture. D. low risk of bronchospasm.

B. low density of the gas mixture.

A 44 week gestational age infant has just been delivered via C-section and is gasping, grunting, and has tachycardia and tachypnea. At one minute his Apgar score is 4 and at 5 minutes the score is 5. The infant is most likely suffering from A. transient tachypnea of the newborn. B. meconium aspiration. C. bronchopulmonary dysplasia. D. apnea of prematurity.

B. meconium aspiration.

A 13 year-old patient in the ED is complaining of dyspnea, chest tightness, and a loose productive cough. The patient has a respiratory rate of 28 breaths/minute and bilateral wheezing in the lungs. What treatment should the respiratory therapist initiate? A. levalbuterol B. oxygen C. salmeterol D. PEP

B. oxygen

The respiratory therapist has obtained a blood gas sample from the patient's radial artery and applied pressure to the site for 10 minutes. After removing any excess air from the syringe, the next step for proper handling of the blood sample is A. adding liquid heparin to the sample. B. placing the syringe in an ice bath. C. shaking the sample continuously. D. applying a pressure bandage.

B. placing the syringe in an ice bath.

The primary source of infection in the health care setting is A. use of medical equipment for multiple patients. B. poor handwashing techniques of personnel. C. food and/or water intake by the patient. D. patient rooms not cleaned appropriately.

B. poor handwashing techniques of personnel.

A patient in the intensive care unit has the following hemodynamic measurements: CVP 4 mm Hg PAP 48/16 mm Hg PCWP 8 mm Hg MAP 92 mm Hg Cardiac Output 5 L/min. Cardiac Index 2.5 L/min/m2 These results are most consistent with A. normal cardiac function B. pulmonary hypertension C. left heart failure D. hypovolemia

B. pulmonary hypertension

A 16-year-old patient involved in a motorcycle accident has been admitted to the emergency department. The paramedic indicates that the patient was not wearing a helmet. To assess the patient's neurologic status the respiratory therapist would assess the patient's A. vital capacity. B. pupillary response. C. herring bruer reflex . D. moro reflexes.

B. pupillary response.

While attempting to calibrate a polarographic oxygen analyzer, the respiratory therapist notices that the analyzer reads 21% when exposed to room air but only reads 64% when exposed to 100% oxygen. The most appropriate action at this time would be to A. reset the zero point. B. replace the battery. C. replace the fuel cell. D. add electrolyte solution.

B. replace the battery.

A spontaneously breathing post-CVA patient has developed right lower lobe infiltrates on chest x-ray and has coarse breath sounds. When the respiratory therapist attempts to suction the patient by the nasotracheal route, she notes a gag reflex is present but the patient does not cough. Watery secretions are aspirated through the suction catheter. The therapist should A. insert an oral endotracheal tube. B. reposition the patient to a sniffing position. C. change to a larger suction catheter. D. insert an oropharyngeal airway.

B. reposition the patient to a sniffing position.

The respiratory therapist is providing patient education for a patient who is being discharged home on aerosol therapy. The most important reason for the patient to follow the recommended cleaning procedures using a vinegar/water solution is that this solution will A. sterilize the equipment. B. retard bacterial growth. C. kill all micro-organisms and spores. D. extend the equipment life.

B. retard bacterial growth.

All of the following could cause a patient's right-hemidiaphragm to be elevated, EXCEPT A. right lower lobe atelectasis. B. right side hyperlucency, absent vascular markings. C. hepatomegaly. D. right lower lobe consolidation with air bronchograms.

B. right side hyperlucency, absent vascular markings

The results of a patient's spirometry are recorded as follows: Predicted Observed FVC (L) 3.5 2.1 FEV1 (L) 2.7 2.6 FRC (L) 2.5 1.7 TLC (L) 5.6 4.2 These results are indicative of A. normal lung function. B. sarcoidosis. C. acute bronchitis. D. cystic fibrosis.

B. sarcoidosis.

The primary reason for the use of respiratory care protocols is to A. decrease the patient workload for therapists. B. standardize provision of care. C. enhance departmental efficiency. D. increase the autonomy of therapists.

B. standardize provision of care.

Which of the following factors are determinants of cardiac output? A. ventricular filling and heart rate B. stroke volume and heart rate C. stroke volume and respiratory rate D. heart rate and tidal volume

B. stroke volume and heart rate

A patient who had his chest tube clamped for 18 hours with no apparent respiratory distress is now tachypneic, diaphoretic and complaining of dyspnea and general uneasiness. The respiratory therapist should A. obtain a chest x-ray. B. unclamp the chest tube. C. increase the suction through the system. D. add water to the suction control bottle.

B. unclamp the chest tube.

The respiratory therapist has been asked to measure Auto-PEEP on a patient receiving mechanical ventilation. In order to do this, the therapist should A.initiate an inspiratory hold just after the next ventilator-delivered breath. B.initiate an expiratory hold just prior to the next ventilator-delivered breath. C.subtract Pplat from Pdyn. D.subtract set PEEP from the measured Pplat.

B.initiate an expiratory hold just prior to the next ventilator-delivered breath.

Which of the following formulas will determine the amount of physiologic deadspace for a patient? A. PAO2 - PaO2 B. CaO2 - CvO2 C. (PaCO2 - PECO2) / PaCO2 D. CcO2 - CaO2 / CcO2 - CvO2

C. (PaCO2 - PECO2) / PaCO2

A pulse oximeter provides an accurate indication of a patient's oxygenation status in which of the following situations? 1. Polycythemia 2. Pulmonary hypertension 3. Congestive heart failure 4. Carbon monoxide poisoning A. 1 & 4 only B. 2 & 3 only C. 1, 2, & 3 only D. 1, 2, 3, & 4

C. 1, 2, & 3 only

What size endotracheal tube would be appropriate for an adult female patient? A. 6.0 to 6.5 mm B. 6.5 to 7.0 mm C. 7.0 to 7.5 mm D. 7.5 to 8.0 mm

C. 7.0 to 7.5 mm

A 30 year old male with bronchitis has coarse bilateral rales with a SpO2 of 90%. Despite a good cough effort, he has great difficulty in removing his thick secretions. The respiratory therapist should initiate A. aerosol treatments with 3 cc normal saline every 4 hours. B. sputum induction for gram stain. C. a heated humidifier and oxygen therapy. D. continuous ultrasonic via aerosol mask.

C. a heated humidifier and oxygen therapy.

A 75 kg (165 lb) male is being mechanically ventilated at the following settings: Mode VC, SIMV Set Rate 12 br/min Total Rate 32 br/min Tidal Volume 600 mL PIP 35 cm H2O Exhaled minute volume 9.2 L FIO2 0.45 PEEP 8 cm H2O The following patient information is available: pH 7.36 PaCO2 45 torr PaO2 89 torr HCO3- 22 mEq/L The respiratory therapist should A. decrease PEEP. B. increase set rate to 14 br/min. C. add pressure support. D. initiate pressure control mode.

C. add pressure support.

The following ABG results are reported for a patient in the ED on room air: pH 7.20; PaCO2 24 torr; PaO2 95 torr; HCO3 8 mEq/L; SaO2 95%; BE -15 mEq/L. The respiratory therapist should recommend A. initiating oxygen therapy via nasal cannula at 4 L/min. B. intubating and initiating mechanical ventilation. C. administering sodium bicarbonate intravenously. D. initiating non-invasive ventilation.

C. administering sodium bicarbonate intravenously.

A post-operative thoracic surgery patient is having difficulty developing an effective cough. The respiratory therapist should recommend all of the following techniques to aid this patient in generating a more effective cough EXCEPT: A. coordinating coughing with pain medication. B. performing serial coughs. C. applying pressure to patient's abdomen during exhalation. D. "splinting" the incision area.

C. applying pressure to patient's abdomen during exhalation.

A 48-year-old woman is admitted to the coronary care unit for chest pain, dizziness and nausea. Her blood pressure is 60/40 mm Hg, respirations are 20/min. The cardiac monitor shows sinus rhythm at a rate of 50/minute with frequent multifocal PVC. The respiratory therapist should recommend administration of: A. lidocaine. B. nitroglycerin. C. atropine. D. amiodarone.

C. atropine.

A pre-op patient who is at high risk for post-operative complications should be evaluated by obtaining the results of A. VD/VT study. B. diffusion studies. C. basic spirometry. D. indirect calorimetry.

C. basic spirometry.

Pre- and post-bronchodilator spirometry is performed on a patient and yields the following results: Pre-Bronchodilator (% predicted) Post-Bronchodilator (% predicted) FVC 82 83 FEV1 46 66 FEF200-1200 51 68 FEF 25-75 49 70 MVV 65 75 The respiratory therapist should recommend the initiation of A. continuous mechanical ventilation B. intermittent positive pressure breathing C. bronchodilator therapy D. incentive spirometry

C. bronchodilator therapy

After consulting on management of a patient with pneumonia and atelectasis, the pulmonologist has documented in the Progress Notes a need to change the patient's treatment regimen. The respiratory therapist should A. continue the current treatment plan. B. report the change in treatment plan to the next shift. C. check the electronic medical record for new physician orders. D. disregard the information until notified by the shift supervisor.

C. check the electronic medical record for new physician orders.

The respiratory therapist notices the low pressure alarm sounding from the ventilator for the patient in ICU Bed 1. As the therapist enters the room, she notes that during inspiration, the pressure manometer reads 6 cm H2O. The therapist's first action should be to A. look for a kink in the circuit. B. change modes of ventilation. C. check the exhalation valve. D. suction the patient.

C. check the exhalation valve.

Pressure-cycled ventilation would be LEAST appropriate for a patient with A. kyphosis. B. muscular dystrophy. C. chronic CO2 retention. D. a drug overdose.

C. chronic CO2 retention.

A 62-year-old female who weighs 60 kg (132 lb) is on mechanical ventilation following hip replacement surgery. Ventilator settings are as follows: Mode VC, SIMV FIO2 0.35 Set rate 12 br/min. Total rate 12 br/min. VT 500 mL Arterial blood gas results are as follows: pH 7.50 PaCO2 30 torr PaO2 105 torr HCO3- 23 mEq/L The respiratory therapist should A. switch to pressure control mode. B. institute pressure support. C. decrease the minute volume. D. decrease the FIO2.

C. decrease the minute volume.

When initiating mechanical ventilation for a patient with chronic CO2 retention, the respiratory therapist must assure adequate inspiratory flow in order to prevent A. a decrease in the patient's FRC. B. an increase in blood pressure. C. dynamic hyperinflation. D. an increase in system pressure.

C. dynamic hyperinflation.

blunt chest trauma, is orally intubated continuous mechanical ventilation is initiated. assessment of the neck and chest reveal a midline trachea & significant reduction in thoracic expansion of left chest. diminished breath sounds in left lung compared to the right lung. most likely indicate which of the following? A. flail chest on right thorax B. right tension pneumothorax C. endobronchial intubation D. subcutaneous emphysema

C. endobronchial intubation

A patient with chronic bronchitis is seen in the pulmonary clinic with complaints of frequent cough and secretion production. Despite completing a round of Keflex® as prescribed, the patient continues to have scattered infiltrates on his chest X-ray. Which of the following tests should the respiratory therapist recommend? A. thoracentesis B. polysomnography C. flexible bronchoscopy D. plethysmography

C. flexible bronchoscopy

A 48 year-old female is admitted to the ED with diaphoresis, jugular venous distension, and 3+ pitting edema in the ankles. These findings are consistent with A. liver failure. B. pulmonary embolism. C. heart failure. D. electrolyte imbalances.

C. heart failure.

A 60 kg (132 lb) female patient with congestive heart failure is receiving NPPV with an IPAP of 16 cm H2O, EPAP of 10 cm H2O, and FIO2 of 0.70. Available laboratory data includes: pH 7.40, PaCO2 42 torr; PaO2 145 torr; HCO3 26 mEq/L, SaO2 99%, CVP 10 cm H2O. Breath sounds reveal a few fine bibasilar crackles. This situation should be described as A. shunting. B. hypoventilation. C. hyperoxygenation. D. fluid overload.

C. hyperoxygenation.

A 75 kg (165 lb) patient with acute lung injury is being mechanically ventilated at the following settings: VC, A/C; VT 300 mL, respiratory rate 16/min, FIO2 0.50 and PEEP +5 cm H2O. The following arterial blood gas results are obtained: pH 7.30, PaCO2 58 torr, PaO2 79 torr, HCO3- 28 mEq/L. The patient's condition should be described as A. venous admixture. B. ventilation/perfusion mismatching. C. hypoventilation. D. refractory hypoxemia.

C. hypoventilation.

A patient has been participating in Pulmonary Rehabilitation for the past 5 weeks. The patient has recorded the results of his 6 minute walk test each week. The results indicate a steady increase in distance over the 5 week period. These results are consistent with A. need to wear supplemental oxygen while walking. B. need for an additional 6 weeks of rehabilitation. C. improvement in exercise ability. D. early discharge from rehabilitation.

C. improvement in exercise ability.

An acceptable level of PEEP therapy can be identified by all of the following EXCEPT A. the lower inflection point of a volume-pressure loop graphic. B. acceptable oxygenation without cardiovascular side effects. C. increasing oxygenation with increasing plateau pressure. D. increasing static compliance with acceptable oxygenation

C. increasing oxygenation with increasing plateau pressure.

What is the most appropriate position for a female patient who is 5'3" tall, weighs 200 kg and is complaining of difficulty breathing? A. Sims B. Trendelenburg C. lateral Fowlers D. Fowlers

C. lateral Fowlers

Bronchial breath sounds heard over the lung periphery indicate A. narrowed airways. B. obstructed bronchi. C. lung consolidation. D. pulmonary edema.

C. lung consolidation.

A 60 year-old male has just been extubated following coronary artery bypass grafting. His chest X-ray demonstrates platelike infiltrates with scattered densities and he is noted to have decreased chest expansion with an increased respiratory rate. Which of the following treatments should be recommended for this patient? A. diuretics B. antibiotics C. lung expansion therapy D. thoracentesis

C. lung expansion therapy

A 62-year-old patient with a history of emphysema (body weight 50 kg, 110 lb) requires mechanical ventilation because of acute respiratory failure. The respiratory therapist notes the following: Mode VC, SIMV Set rate 10 br/min. Total rate 16 br/min VT 400 mL FIO2 0.28 Arterial blood gases are as follows: pH 7.37 PCO2 56 torr PO2 65 torr HCO3- 32 mEq/L The respiratory therapist should A. increase the set rate to 8 br/min. B. increase the FIO2 to 0.30. C. maintain the current settings. D. increase the VT to 650 mL.

C. maintain the current settings.

While reviewing the results of polysomnography on a 44-year-old patient, the respiratory therapist notes that the Apnea Hypopnea Index (AHI) is 25. This value should be reported as A. normal. B. mild. C. moderate. D. severe.

C. moderate.

A 60 year-old male is admitted to the ED with chest pain. The CBC and electrolytes are normal. Troponin level is 0.4 ng/mL. The physician should report to the patient that he is suffering from A. pulmonary embolism. B. gastroesophageal reflux. C. myocardial infarction. D. valvular stenosis.

C. myocardial infarction.

During a pre-operative evaluation, bedside spirometry results are as follows: FVC 88% of predicted, FEV1 85% of predicted, FEV1/FVC 82% of predicted and FEF25-75 81% of predicted. How should the respiratory therapist interpret these results? A. a mild restrictive disorder B. a mild obstructive disorder C. normal lung function D. mixed obstructive/restrictive disorder

C. normal lung function

Over the last hour a patient being ventilated with a volume cycled ventilator has had a decrease in urine output from 35 mL/hour to 10 mL/hour. The most likely cause of this change would be an increase in the A. respiratory rate. B. delivered FIO2 . C. peak airway pressure. D. inspiratory flow.

C. peak airway pressure.

All of the following could cause the high pressure alarm on a volume cycled ventilator to sound EXCEPT A. decrease in lung compliance. B. increase in airway resistance. C. peak flow setting of 75 L/min. D. acute bronchospasm.

C. peak flow setting of 75 L/min.

A patient who complains of dyspnea is noted to have a dry, non-productive cough. On physical examination, breath sounds are diminished on the right, tactile fremitus is decreased and there is dullness to percussion over the right lower lobe. The respiratory therapist should suspect that the patient is suffering from A. pneumonia. B. pulmonary embolism. C. pleural effusion. D. bronchiolitis.

C. pleural effusion.

The respiratory therapist is instructing a postoperative abdominal surgery patient on how to perform incentive spirometry. The therapist should explain that the purpose of the treatment is to A. prevent microatelectasis. B. promote healing of the incision. C. prevent areas of lung collapse. D. prevent and treat pneumonia.

C. prevent areas of lung collapse.

A tracheostomy tube has just been changed on a patient receiving continuous volume-cycled ventilation. The patient suddenly becomes dyspneic and develops crepitus around the tracheostomy stoma. The respiratory therapist should A. suction the patient. B. insert a larger tracheostomy tube. C. reposition the tracheostomy tube. D. recommend an antihistamine.

C. reposition the tracheostomy tube.

A 19-year-old patient has a size 7 mm ID nasotracheal tube placed through the right nare. While attempting to suction the patient with a size 12 Fr catheter, resistance is met as the catheter is introduced into the endotracheal tube. The respiratory therapist should A. lubricate the catheter with petroleum jelly prior to suctioning. B. instill acetylcysteine into the endotracheal tube. C. switch to a 10 Fr suction catheter. D. extubate the patient and reintubate with a size 9 mm ID endotracheal tube.

C. switch to a 10 Fr suction catheter.

The following measurements were obtained from a patient being monitored with a balloon tipped, flow directed catheter: CVP 2 cmH2O PAP 12 torr (mean) PCWP 3 torr CI 1.2 L/min/m2 BP 110/90 mm Hg Based on the above information, all of the following parameters would be decreased EXCEPT A. right ventricular end diastolic pressure. B. left atrial filling pressure. C. systemic vascular resistance. D. cardiac output.

C. systemic vascular resistance.

A patient who is receiving continuous mechanical ventilation is fighting the ventilator. His breath sounds are markedly diminished on the left, there is dullness to percussion on the left, and the trachea is shifted to the left. The most likely explanation for the problem is that A. the patient is disconnected from the ventilator. B. the patient is experiencing diffuse bronchospasm. C. the endotracheal tube has slipped into the right main stem bronchus. D. the patient has developed a left tension pneumothorax.

C. the endotracheal tube has slipped into the right main stem bronchus.

A patient is receiving oxygen via nasal cannula at 2 L/min and has the following ABG results: pH 7.37, PaCO2 42 torr, PaO2 80, HCO3 38 mEq/L. The most likely explanation for these results is that A. the sample was not iced properly. B. there was excess heparin in the syringe. C. the numbers were not reported correctly. D. The sample contains venous blood.

C. the numbers were not reported correctly.

A 33 year-old patient with trauma has been ventilated at the current settings for 24 hours. While reviewing ventilator data from the patient, the respiratory therapist notes the following: 0800 1000 1200 Peak Inspiratory Pressure (cm H2O) 28 35 50 Plateau Pressure (cm H2O) 23 25 25 This information would indicate that A. airway resistance is decreasing. B. lung compliance is increasing. C. the patient needs suctioning. D. the patient can begin weaning.

C. the patient needs suctioning.

The respiratory therapist prepares to assist with a bronchoscopy of a patient in the ICU currently receiving mechanical ventilation in the VC,AC mode. The therapist should anticipate addressing all of the following considerations EXCEPT A. introduction of the bronchoscope will create increased resistance to flow. B. the tidal volume of the patient must be closely monitored during the procedure. C. the patient's extrinsic PEEP levels will increase. D. peak inspiratory pressure on the ventilator will rise.

C. the patient's extrinsic PEEP levels will increase.

The physician informs the patient that the results of his polysomnogram indicate obstructive sleep apnea. Treatment for this disorder might include A. bronchial hygiene therapy. B. respiratory stimulants. C. tracheostomy. D. negative pressure ventilation.

C. tracheostomy.

The FRC measured by body plethysmography is 30% larger than that measured by helium dilution. This difference is best explained by A. maldistribution of ventilation. B. increased diffusing capacity. C. trapped thoracic gas. D. reduced lung compliance.

C. trapped thoracic gas.

A patient has mild stridor immediately after extubation. This finding is most often associated with A. lower airway obstruction. B. secretions in the large airways. C. upper airway obstruction. D. bronchial spasm.

C. upper airway obstruction.

All of the following strategies are likely to decrease the likelihood of damage to the tracheal mucosa EXCEPT A. maintaining cuff pressures between 20 and 25 mm Hg. B. using the minimal leak technique for inflation. C. using a low-residual-volume, low-compliance cuff. D. monitoring intracuff pressures.

C. using a low-residual-volume, low-compliance cuff.

An intubated patient receiving 30% oxygen has a SpO2 of 80% and ETCO2 of 40 torr. After administration of 50% oxygen for 30 minutes, the respiratory therapist notes that the SpO2 rises to 98% and the ETCO2 remains stable at 40 torr. The major cause of hypoxemia in this patient is A. hypoventilation. B. shunt. C. ventilation/perfusion mismatch. D. increased deadspace.

C. ventilation/perfusion mismatch.

A 16 year-old patient with cystic fibrosis attends public high school. Which of the following bronchial hygiene therapies would be most appropriate for this patient? A. intrapulmonary percussive ventilation B. dornase alpha therapy C. vibratory / oscillatory PEP D. postural drainage and manual percussion

C. vibratory / oscillatory PEP

A respiratory therapist is calibrating a thermal conductivity helium analyzer. What should the analyzer read when exposed to room air? A. 100% B. 79% C. 21% D. 0%

D. 0%

A patient is on CPAP at 10 cmH2O and 0.30 FIO2 with the heated humidifier set at 40° C. As the gas is delivered to the patient through large bore tubing, which of the following will occur? 1. Excess water will rain out 2. Humidity deficit will occur 3. Relative humidity will decrease 4. Relative humidity will remain 100% A. 2 only B. 2 and 4 only C. 1 and 3 only D. 1 and 4 only

D. 1 and 4 only

After assisting with bronchoalveolar lavage and lung biopsy on a mechanically ventilated patient, the respiratory therapist notes the activation of a high pressure alarm. Peak inspiratory pressure has increased from 32 cm H2O before the procedure to 45 cm H2O after the procedure. Possible causes for the increased pressure include 1. bronchospasm. 2. pneumothorax. 3. pulmonary hemorrhage. A. 1 and 2 only B. 1 and 3 only C. 2 and 3 only D. 1, 2, and 3

D. 1, 2, and 3

The respiratory therapist is preparing to administer inhaled nitric oxide to a neonate with respiratory distress syndrome. The most appropriate initial dose of iNO for this patient is A. 5 ppm. B. 10 ppm. C. 15 ppm. D. 20 ppm.

D. 20 ppm

What is normal urine output in an adult patient? A. 10 mL/hr B. 20 mL/hr C. 30 mL/hr D. 40 mL/hr

D. 40 mL/hr

A patient on VC, SIMV with a VT of 500 mL has a PIP of 25 cm H2O, Pplat of 15 cm H2O and PEEP of 5 cm H2O. What is the patient's static lung compliance? A. 25 mL/cm H2O B. 35 mL/cm H2O C. 45 mL/cm H2O D. 50 mL/cm H2O

D. 50 mL/cm H2O

Which of the following values for arterial carbon dioxide tension is consistent with significant alveolar hypoventilation? A. 20 torr B. 30 torr C. 40 torr D. 50 torr

D. 50 torr

At 1 minute post-delivery, a newborn has blue extremities with a pink body, heart rate is 90/min, respiratory rate is 20/min with a weak cry, cough reflex is present, and there is some flexion of the extremities. At 5 minutes post-delivery, the infant is completely pink, heart rate is 140/min, respiratory rate is 40/min, cough reflex is present, and the baby is active with a strong cry. What APGAR scores should be assigned? A. 4 & 8 B. 5 & 9 C. 5 & 10 D. 6 & 10

D. 6 & 10

While reviewing quality control data for the blood gas lab, the respiratory therapist notes the following data plot for the pH electrode: Which of the following should the therapist recommend? A. Repeat the previous control analysis B. Remove the analyzer from service C. Reset the analyzation module on the analyzer D. Recalibrate the pH electrode

D. Recalibrate the pH electrode

A 52 year-old post-operative cholecystectomy patient's breath sounds become more coarse upon completion of postural drainage with percussion. The respiratory therapist should recommend A. continuing the therapy until breath sounds improve. B. administering dornase alpha. C. administering albuterol therapy. D. deep breathing and coughing to clear secretions.

D. deep breathing and coughing to clear secretions.

A patient is diagnosed with unilateral lung disease. The physician has requested that the effected lung be ventilated at a pressure 10 cm H2O lower than the normal lung. The respiratory therapist should recommend providing this type of ventilation via A. cricothyroidotomy. B. esophageal combi-tube. C. transtracheal catheter. D. double-lumen endobronchial tube.

D. double-lumen endobronchial tube.

While making oxygen rounds, the respiratory therapist hears a high-pitched sound coming from a bubble humidifier. The patient is receiving oxygen by air-entrainment mask at 28% and the oxygen flowmeter is set at 12 L/min. The therapist should A. replace the cracked humidifier. B. increase the FIO2 on the air-entrainment mask. C. increase the flow from the flowmeter. D. eliminate the bubble humidifier.

D. eliminate the bubble humidifier

While providing patient education to patients who will be discharged home on oxygen therapy, the respiratory therapist explains the hazards associated with oxygen delivery equipment in the home. This instruction should include all of the following EXCEPT A. liquid oxygen burns when refilling portable tanks. B. how to properly secure oxygen cylinders for transport. C. use of grounded 3-prong outlets for electrical equipment. D. emergency procedure to deal with gas explosions.

D. emergency procedure to deal with gas explosions.

A patient with end-stage pulmonary fibrosis is receiving oxygen at 2 L/min via a transtracheal oxygen catheter. The patient experiences an increased work of breathing and shortness of breath. The respiratory therapist should A. manually ventilate the patient with a resuscitation bag. B. increase the flow to the transtracheal catheter to 6 L/min. C. evaluate the SpO2 with a pulse oximeter. D. flush the transtracheal device with isotonic saline.

D. flush the transtracheal device with isotonic saline.

A 36 year-old fireman was trapped and subsequently rescued from the collapse of a burning building. Which of the following devices would be appropriate to accurately assess his oxygenation status? A. capnograph B. pulse oximeter C. blood gas analyzer D. hemoximeter

D. hemoximeter

A 72-year-old male patient who is 5' 10" tall and weighs 75 kg (165 lb) is receiving mechanical ventilation. The respiratory therapist notes diminished breath sounds in the bases of both lungs. The patient is on the following ventilator settings: Mode VC, A/C VT 600 mL Set rate 10 br/min PEEP 10 cm H2O FIO2 0.50 Sigh volume 0.6 L Arterial blood gases show: pH 7.42 PaO2 82 torr PaCO2 38 torr SaO2 93% Based on this information, the respiratory therapist should A. maintain the current settings. B. increase the set rate to 12 br/min. C. decrease the tidal volume to 500 mL. D. increase the sigh volume to 1.0 L.

D. increase the sigh volume to 1.0 L.

A spontaneous breathing trial has been initiated for a 64-year-old, intubated, post-op patient. Oxygen is being administered by T-piece at an FIO2 of 0.50 via heated air-entrainment nebulizer at a flow of 12 L/min and a temperature of 37.0oC. The respiratory therapist notices that the aerosol mist disappears from the reservoir outlet during the patient's inspiration. The therapist should consider all the following EXCEPT A. lenghtening the reservoir tubing. B. adding an additional nebulizer. C. increasing the flow to 15 L/min. D. increasing the temperature to 39C.

D. increasing the temperature to 39.0oC.

A 28 year-old female has just been admitted through the ED with suspected CO poisoning. She is receiving oxygen by non-rebreather mask at 10 L/min. Upon entering the patient's room, the respiratory therapist notes that the reservoir bag of the mask collapses during inspiration. This is most likely the result of A. faulty one-way valves. B. tight seal between the mask and the patient's face. C. presence of a bubble humidifier. D. insufficient flow to the reservoir bag.

D. insufficient flow to the reservoir bag.

After performing spirometry on a patient in the pulmonary clinic, the respiratory therapist notes that both the inspiratory and expiratory flow portion of the flow-volume loop is flattened. The therapist should interpret the condition demonstrated on the flow-volume loop as a/an A. normal tracing. B. obstructive pattern. C. restrictive pattern. D. large airway obstruction.

D. large airway obstruction.

Evaluation of a spontaneously breathing patient reveals tachypnea, tracheal deviation to the right and an absence of breath sounds on the left. The most likely etiology would be A. bronchiectasis. B. myasthenia gravis. C. acute asthmatic attack. D. left tension pneumothorax.

D. left tension pneumothorax.

The most common complication associated with fiberoptic bronchoscopy via the nasal route is A. nosocomial infection. B. hypoxemia. C. uncontrolled coughing. D. mild epistaxis.

D. mild epistaxis.

A 72 year-old female post stem cell transplant patient in the ICU is complaining of difficulty breathing and is noted to have diffuse fluffy infiltrates on chest X-ray. The B-type Natriuretic Peptide (BNP) test result demonstrates 700 pg/mL. What is the patient's possible condition? A. severe heart failure B. respiratory distress syndrome C. severe renal failure D. moderate heart failure

D. moderate heart failure

Airways resistance (Raw) of 1.8 cm H2O/L/sec is measured for a patient receiving mechanical ventilation. The respiratory therapist should suspect that the patient may have A. asthma. B. ARDS. C. neuromuscular disease. D. normal airways.

D. normal airways.

In order to reduce a patient's PaCO2 from 40 torr to 32 torr, all of the following could be increased EXCEPT A. tidal volume. B. alveolar ventilation. C. respiratory rate. D. physiologic deadspace

D. physiologic deadspace

A patient recently underwent a coronary artery bypass and is now receiving Incentive Spirometry QID. The patient's inspiratory capacity has diminished over the past several days and the patient is becoming increasingly short of breath. Chest radiograph shows left lower lobe consolidation with air bronchograms. The respiratory therapist would conclude that the patient has developed A. pleural effusion. B. atelectasis. C. pneumothorax. D. pneumonia.

D. pneumonia.

The most common hazard associated with insertion of a central venous catheter is A. pulmonary embolus. B. hypoxemia. C. rupture of the right atrium. D. pneumothorax.

D. pneumothorax.

All of the following conditions can be treated with hyperbaric oxygen (HBO) therapy EXCEPT A. carbon monoxide poisoning. B. decompression sickness. C. anaerobic infections. D. pulmonary hypertension.

D. pulmonary hypertension.

A patient receiving mechanical ventilation has a capnometer in-line at the Y-connector of the vent circuit for continuous monitoring of exhaled CO2. The capnogram suddenly indicates an abrupt decrease in the PETCO2 from 5.3% to 0.0%. The respiratory therapist should A. replace the exhalation valve. B. decrease the humidifier temperature setting. C. remove the inline medication nebulizer. D. reattach the patient to the circuit.

D. reattach the patient to the circuit.

Immediately after extubation of a patient in the ICU, the respiratory therapist observes increasing respiratory distress with intercostal retractions and marked stridor. The SpO2 on 40% oxygen is noted to be 86%. Which of the following would be most appropriate at this time? A. cool mist aerosol treatment B. aerosolized racemic epinephrine C. manual ventilation with resuscitation bag and mask D. reintubation

D. reintubation

A nasally intubated patient is receiving mechanical ventilation via volume-cycled ventilator. While performing a patient-ventilator system check, the respiratory therapist notices that the high pressure alarm is sounding. The patient appears cyanotic and his heart rate is 48 beats per minute. The therapist is unable to pass a suction catheter through the nasotracheal tube. The therapist should A. increase the FIO2 to 1.00. B. call the patient's physician. C. obtain a stat arterial blood gas. D. remove the tube and manually ventilate the patient.

D. remove the tube and manually ventilate the patient.

While weaning a patient from inhaled nitric oxide therapy, the respiratory therapist notes an increase in pulmonary artery pressure following a recent decrease in dosage. The therapist should A. discontinue the nitric oxide. B. call the physician for guidance. C. continue to monitor the patient. D. return the iNO dose to the previous level.

D. return the iNO dose to the previous level.

The respiratory therapist instructs a patient to take a maximal inspiration followed by a maximal exhalation without force. Which of the following values are being measured? A. expiratory reserve volume B. residual volume C. functional residual capacity D. slow vital capacity

D. slow vital capacity

The respiratory therapist has just assisted the pulmonologist with a bedside fiberoptic bronchoscopy procedure in the ICU. In order to clean and disinfect the bronchoscope, the therapist should A. rinse with sterile water and steam autoclave for 15 minutes. B. wipe with Betadine and pasteurize for 30 minutes. C. sterilize with ethylene oxide. D. soak in alkaline glutaraldehyde for 10 hours.

D. soak in alkaline glutaraldehyde for 10 hours.

The respiratory therapist is preparing to assist in the intubation of an adult male patient. The anesthesia resident wishes to administer a neuromuscular blocker that has a fast onset and short duration. Which of the following should the therapist recommend? A. ketamine B. vecuronium C. rocuronium D. succinylcholine

D. succinylcholine

A home care patient calls in the middle of the night and reports that the oxygen supply tubing will not stay attached to her transtracheal catheter. The flow rate to the transtracheal catheter is set at 0.5 L/min. The patient has attempted to flush the catheter with saline and push a cleaning rod through it without success. The respiratory therapist should instruct the patient to A. tape the connection securely. B. increase the flow to the catheter. C. decrease the flow to the catheter. D. switch to a nasal cannula.

D. switch to a nasal cannula.

A 55 year-old male patient is being evaluated for pulmonary rehabilitation. During a cycle ergometer cardiopulmonary stress procedure, the patient has a heart rate of 100/min and a respiratory rate of 20/min. He suddenly begins to complain of chest pain and severe shortness of breath. The respiratory therapist should A. reduce the speed of the bike. B. administer supplemental oxygen. C. gradually reduce the workload and monitor closely. D. terminate the procedure immediately.

D. terminate the procedure immediately.

Which of the following formulas will determine the total flow being delivered to a patient with a 28% venturi mask running at 6 L/min? A. total flow = 6 x 2 B. total flow = 6 x 4 C. total flow = 6 x 5 D. total flow = 6 x 11

D. total flow = 6 x 11

A patient with a closed head injury has had a cuffed tracheostomy tube in place for several weeks. The physician wishes to decannulate the patient but maintain the patency of the stoma for secretion removal. Which of the following devices would facilitate this request? A. fenestrated trach tube B. transtracheal catheter C. laryngectomy tube D. tracheostomy button

D. tracheostomy button

A 50 kg (110 lb) patient is being mechanically ventilated with the following settings: VC, A/C, VT 400 mL, respiratory rate 14/min, FIO2 0.60 and 10 cm H2O PEEP. The chest radiograph demonstrates diffuse bilateral radiopacity. ABG results are: pH 7.36, PaCO2 47 torr, PaO2 50 torr, and HCO3- 28 mEq/L. The respiratory therapist should increase the A. PEEP. B. FIO2. C. expiratory time. D. respiratory rate

A. PEEP

Which of the following would indicate that the lung compliance of a patient on a volume-cycled ventilator is increasing? A. Plateau pressure begins to decrease B. Peak pressure begins to increase C. Blood pressure decreases D. Arterial PCO2 decreases

A. Plateau pressure begins to decrease

A patient develops ascites and shortness of breath. Where is tissue edema most likely to show up first? A. Ankles B. Abdomen C. Thorax D. Hands

B. Abdomen

A newborn infant has an APGAR score of 8 one minute after delivery. The most appropriate action at this time would be to A. intubate and ventilate the infant. B. dry and monitor the infant. C. initiate oxygen therapy. D. begin chest compressions immediately.

B. dry and monitor the infant.

During oral endotracheal intubation, the tip of the Macintosh laryngoscope blade should be placed A. between the soft palate and tongue. B. in the vallecula. C. under the epiglottis. D. between the vocal cords.

B. in the vallecula.

A 7 year old child suspected of having epiglottitis would exhibit which of the following signs? 1. Drooling 2. Hyperextended neck 3. Stridor 4. Unilateral wheeze A. 1,3, B. 4 C. 1, 2, 3 D. 2, 4

C. 1, 2, 3

Which of the following suction catheters would be appropriate to use for a patient with a size 8.0 mm ID endotracheal tube? A. 8 Fr B. 10 Fr C. 12 Fr D. 14 Fr

C. 12 Fr

An adult patient with asthma is receiving a mixture of 70% helium and 30% oxygen through a nonrebreathing mask with an oxygen flowmeter set at 10 L/min. What is the actual flow being delivered to the mask? A. 10 L/min B. 13 L/min C. 16 L/min D. 18 L/min

C. 16 L/min

A patient in the ICU receiving mechanical ventilation has just undergone a fiberoptic bronchoscopy procedure in which a tissue biopsy was collected. Immediately following the procedure, the respiratory therapist notes that the peak inspiratory pressure on the ventilator has increased. Potential causes for this include all of the following EXCEPT A. hypoxemia. B. pneumothorax. C. pulmonary hemorrhage. D. bronchospasm/laryngospasm.

A. hypoxemia.

The most serious complication associated with airway suctioning is A. hypoxemia. B. bradycardia. C. mucosal trauma. D. gag reflex stimulation

A. hypoxemia.

Which of the following airway clearance techniques uses a pneumatic device to deliver compressed gas mini-bursts at sub-tidal volumes to the airway at frequencies of 100 to 250/min? A. intrapulmonary percussive ventilation B. autogenic drainage C. high-frequency chest wall compression device D. positive expiratory pressure breathing

A. intrapulmonary percussive ventilation

Which of the following short-acting beta agonists can be delivered via aerosol therapy? A. levalbuterol (Xopenex®) B. tiotropium (Spiriva®) C. dornase alpha (Pulmozyme®) D. salmeterol (Serevent®)

A. levalbuterol (Xopenex®)

A patient receiving a loop diuretic such as furosemide (Lasix) would most likely need what type of electrolyte replacement? A. Potassium B. Sodium C. Chloride D. Bicarbonate

A. Potassium

A 77-year-old male patient is admitted to the emergency room with shortness of breath, fine basilar crackles, +2 pitting edema and a chest radiogram with a butterfly pattern. These results are most consistent with which of the following? A. Pulmonary edema B. Pulmonary interstitial emphysema C. Pneumothorax D. Emphysema

A. Pulmonary edema

While administering 3.5 mg of albuterol to a patient with asthma in the ICU, the respiratory therapist notes that the patient's heart rate increases from 120 to 150 beats/minute. What is the appropriate modification for the next treatment for this patient? A. Reduce the dose of albuterol. B. Discontinue the treatment. C. Change to 3 puffs of beclomethasone dipropionate (Vanceril®). D. Change to 0.63 mg of levalbuterol (Xopenex®).

A. Reduce the dose of albuterol.

A patient is receiving oxygen at home via nasal cannula at 1 L/min. He has 50 feet of extension tubing attached to his oxygen concentrator. The patient complains that there does not seem to be enough oxygen flow reaching the cannula. The respiratory therapist should recommend A. decreasing the length of the extension tubing. B. increasing the concentrator flow. C. changing to a liquid system. D. analyzing the FIO2.

A. decreasing the length of the extension tubing.

A patient is being ventilated with a high frequency ventilator. The results of an arterial blood gas analysis are as follows: pH 7.29 PaCO2 68 torr PaO2 73 torr HCO3- 25 mEq/L BE +1 mEq/L The respiratory therapist should increase the A. driving pressure. B. frequency. C. bias flow. D. I:E ratio.

A. driving pressure.

A patient receiving warfarin (Coumadin®) has a prothrombin time (PT) of 20 seconds. These findings indicate a A. high likelihood of excessive bleeding. B. normal clotting ability. C. propensity for increased clotting. D. decrease in bone marrow function.

A. high likelihood of excessive bleeding.

Which of the following would be the most appropriate test to evaluate partial vocal cord paralysis in a patient complaining of difficulty swallowing? A. SB nitrogen elimination B. Maximum voluntary ventilation C. Flow volume loop D. Diffusing capacity

C. Flow volume loop

Which of the following techniques measures total lung capacity? 1. Helium dilution 2. Body plethysmography 3. Single breath nitrogen elimination A. 1 &2 B. 2 only C. 3 only D. 1 & 3

A. 1 &2

Which of the following information may be obtained from a FVC maneuver during bedside pulmonary function testing? 1. FEV1 2. PEFR 3. FRC 4. RV A. 1 and 2 only B. 1 and 3 only C. 2 and 4 only D. 3 and 4 only

A. 1 and 2 only

Which of the following should the respiratory therapist consider when preparing for helicopter transport of a patient receiving mechanical ventilation? 1. Select a ventilator that uses demand valves rather than a reservoir IMV system. 2. Calculate oxygen cylinder duration of flow. 3. Selecting a ventilator that incorporates an internal air compressor. A. 1 and 2 only B. 2 and 3only C. 3 only D. 1, 2, and 3

A. 1 and 2 only

How long will a full E cylinder of oxygen last if run until empty with a flowrate of 10 L/min? A. 1 hour B. 10 hours C. 33 hours D. 66 hours

A. 1 hour

A mixed venous blood sample is needed to determine the oxygen consumption of the tissues. The mixed venous blood sample should be obtained from the A. left atrium. B. pulmonary vein. C. pulmonary artery. D. superior vena cava.

C. pulmonary artery.

A patient on VC ventilation has demonstrated auto-PEEP on ventilator graphics. Which of the following controls, when adjusted independently, would increase expiratory time? 1. Tidal volume 2. Respiratory Rate 3. Inspiratory flow 4. Sensitivity A. 1, 2, and 3 only B. 1, 2, and 4 only C. 1, 3, and 4 only D. 2, 3, and 4 only

A. 1, 2, and 3 only

While suctioning a patient who is being mechanically ventilated, the respiratory therapist notes the following ECG pattern on the monitor: Suctioning is stopped and the patient is returned to the ventilator. Which of the following drugs should the respiratory therapist recommend FIRST? A. 100% oxygen B. atropine C. lidocaine D. epinephrine

A. 100% oxygen

What is the air-to-oxygen ratio for an air entrainment device delivering 60% oxygen? A. 1:1 B. 3:1 C. 4:1 D. 10:1

A. 1:1

What is the normal range for central venous pressure in an adult? A. 2 - 6 mm Hg B. 4 - 12 mm Hg C. 9 - 18 mm Hg D. 21 - 28 mm Hg

A. 2 - 6 mm Hg

During chart review prior to obtaining an ABG sample, the respiratory therapist notes that the patient has a platelet count of 115,000/mm3. Based on this finding, what should the therapist do? 1. Perform ABG as normal. 2. Refuse to perform the ABG. 3. Hold pressure on the puncture site for a longer time after sample is collected. 4. Recommend that an ABG should be performed on the patient only if absolutely necessary. A. 2 and 4 only B. 1 and 3 only C. 3 and 4 only D. 2 only

A. 2 and 4 only

Prior to performing a cardiopulmonary stress test on a 60-year-old patient, the respiratory therapist determines the incremental workloads to be used for test. What is the patient's maximum heart rate? A. 100 beats per minute. B. 130 beats per minute. C. 160 beats per minute. D. 190 beats pe minute.

C. 160 beats per minute.

A patient with bilateral pneumonia is receiving mechanical ventilation in the intensive care unit. The following data is obtained: Mode PC Set rate 12 b/min Total rate 12 b/min VE 8.6 L FIO2 0.60 PIP 31 cm H2O PEEP 10 cm H2O pH 7.41 PaCO2 40 torr PaO2 95 torr SaO2 96% HCO3- 23 mEq/L BE +1 mEq/L PAO2 370 torr C(a-v)O2 3.5 vol% The respiratory therapist should report the A-aDO2 as A. 275 torr. B. 300 torr. C. 345 torr. D. 370 torr.

A. 275 torr.

While performing a 12-lead electrocardiograph on a 68-year-old male patient, the respiratory therapist notices a sudden increase in artifact on the ECG paper. Upon further investigation, the therapist discovers that the V2 electrode has fallen off the chest. Where on the patient should the therapist place this electrode? A. 4th intercostal space on the left side of the sternum. B. 5th intercostal space, left mid-clavicular line. C. 6th intercostal space, left mid-axillary line. D. 7th intercostal space, right mid-clavicular line.

A. 4th intercostal space on the left side of the sternum.

The respiratory therapist is completing oxygen rounds on the ward and checking oxygen saturations on a number of patients. What solution would be most appropriate for disinfecting the surface of the pulse oximeter between patients? A. 70% ethyl alchohol B. Warm soapy water C. Bleach D. Acid glutaraldehyde

A. 70% ethyl alchohol

Which of the following would modify the expiratory time and change the I:E ratio for a patient receiving IPPB therapy? A. Expiratory resistance valve B. Inspiratory flow C. Rate control D. Volume control

A. Expiratory resistance valve

While performing diagnostic chest percussion, the respiratory therapist notes decreased resonance to percussion. Which of the following are potential causes of this finding? 1. pneumothorax 2. pleural effusion 3. pneumonia 4. atelectasis A. 1 and 3 only B. 2 and 4 only C. 2, 3, and 4 only D. 1, 2, 3, and 4

C. 2, 3, and 4 only

A patient is being evaluated in the pulmonary clinic. He reports that he smoked a pack and a half of cigarettes (30 cigarettes) per day for 20 years. How should the respiratory therapist describe the patient's smoking history? A. 20 pack-years B. 25 pack-years C. 30 pack-years D. 60 pack-years

C. 30 pack-years

A patient is receiving 40% oxygen via a Venturi mask at 8 L/min. While performing oxygen rounds, the respiratory therapist notes that the flowmeter setting has been changed to 12 L/min. How would this change affect the accuracy of this device? A. FIO2 will remain unchanged B. FIO2 will decrease to 0.35 C. air entrainment factor will decrease D. FIO2 will increase to 0.50

A. FIO2 will remain unchanged

A 55 year-old post cardiac surgery patient has the following ABG results: pH 7.50, PaCO2 30 torr, PaO2 62 torr, HCO3 25 mEq/L, SaO2 92%, HB 14 g/dL, BE +2. Venous blood gas results are pH 7.39, PvCO2 43 torr, PvO2 37 torr, and SvO2 66%. Calculate the patient's C(a-v)O2. A. 2.5 vol% B. 4.0 vol% C. 5.0 vol% D. 5.5 vol%

C. 5.0 vol%

A post-operative thoracotomy patient is receiving incentive spirometry therapy Q2H. Breath sounds are diminished in the bases of the lungs with scattered crackles. The patient's inspiratory capacity has decreased over the past 2 days. A chest radiograph indicates thin-layered basilar densities. Which of the following has most likely occurred? A. Atelectasis B. Pneumonia C. Pulmonary edema D. Consolidation

A. Atelectasis

Which of the following is an indication for high frequency jet ventilation? A. Bronchopleural fistula B. Wilson Mikity syndrome C. Necrotizing lesion of right lung D. Centrilobular emphysema

A. Bronchopleural fistula

For a patient enrolled in a smoking cessation program, which of the following would indicate that the patient is non-compliant? A. COHb level of 7% B. Regular attendance at counseling sessions C. Increased exercise tolerance D. Decreased use of accessory muscles

A. COHb level of 7%

If a patient's PvO2 decreased from 30 torr to 20 torr, which of the following should the respiratory therapist assess? A. Cardiac output B. Venous blood gas C. Capillary wedge pressure D. Pulmonary vascular resistance

A. Cardiac output

Which of the following will give the most accurate measurement of volume and flow for spirometry? A. Collins water-sealed spirometer B. Vortex-shedding pneumotachometer C. Wright respirometer D. Dry-rolling spirometer

A. Collins water-sealed spirometer

The physician asks the respiratory therapist to set ventilator parameters that will deliver the lowest peak inspiratory pressure possible. Which of the following inspiratory flow patterns will enable the therapist to fulfill the physician's request? A. Decelerating B. Square wave C. Constant D. Accelerating

A. Decelerating

All of the following are TRUE statements about spacers and holding chambers, EXCEPT A. Do not require patient cooperation with their breathing pattern. B. Improve the efficiency of MDI. C. Can be used for drug delivery by MDI to intubated and mechanically ventilated patients. D. If a patient exhales immediately following activation of the inhaler, they will clear the medication from the device and waste the dose

A. Do not require patient cooperation with their breathing pattern.

During recovery from resection of an aortic aneurysm, a 65-year-old female patient suddenly develops severe substernal chest pain with grave dyspnea. The physician describes the bilateral breath sounds as basilar moist crepitant crackles. The patient appears pale, cool and diaphoretic. Which of the following should the respiratory therapist recommend as part of the initial assessment of this patient? A. ECG B. CBC C. Serum electrolytes D. Lateral decubitus radiograph

A. ECG

A tracheostomy tube has just been inserted percutaneously into a patient with a C3 fracture. How much air should the respiratory therapist initially inject into the cuff? A. Enough to achieve a pressure of 25-35 cmH2O. B. Enough to achieve a minimal occluding volume. C. A minimum of 20 mL. D. Until firm tension is felt in the pilot balloon.

A. Enough to achieve a pressure of 25-35 cmH2O.

A patient with chronic hypercapnia is brought into the ED. The history of the present illness revealed that the patient passed out at home. A pulmonary artery catheter has been placed with the following measurements obtained: PAP 25/10 mm Hg BP 76/50 mm Hg PCWP 4 mm Hg SVR 1360 dynes CVP 0 mm Hg CI 1.8 L/min/m2 Which of the following is the most likely cause for his condition? A. Hypovolemia B. Drug overdose C. Cor pulmonale D. High FIO2 vasodilation

A. Hypovolemia

Which of the following pulmonary function measurements would have the smallest predicted normal value? A. IRV B. TLC C. VC D. IC

A. IRV

A patient is receiving mechanical ventilation at the following settings: Mode SIMV Mandatory rate 12 Total rate 12 FIO2 0.50 VT 700 mL Peak flow 50 L/min PEEP 5 cm H2O The following volume-pressure loop is observed. Which of the following should the respiratory therapist recommend? A. Increase the PEEP B. Change to assist-control mode C. Increase the VT D. Increase the peak flow

A. Increase the PEEP

Which of the following findings is LEAST compatible with hyperlucency as seen on a chest x-ray? A. Increased fremitus B. Decreased intensity of breath sounds C. Diminished diaphragmatic excursion D. Hyperresonance to percussion

A. Increased fremitus

A patient in the intensive care area has the following laboratory data: pH 7.54 PaCO2 48 torr PaO2 78 torr CaO2 18.9 vol% K+ 2.1 mEq/L Cl- 82 mEq/L Na+ 142 mEq/L Based on these results, the respiratory therapist should recommend administration of A. KCl. B. NaHCO3. C. NaCl. D. O2.

A. KCl.

The following pulmonary function data was reported for a 45 year old pre-op patient: TLC - 5.4 L RV - 1.0 L IRV - 2.6 L VC - 4.2 L ERV - 1.0 L VT - 0.6 L FRC - 2.0 L IC - 3.2 L Which of the above capacities is incorrect? A. TLC B. VC C. FRC D. IC

A. TLC

A patient has a tracheostomy tube in place. The measured cuff pressure is 24 mm Hg. Which of the following statements is true of this situation? A. The pressure is appropriate for minimal tracheal occlusion. B. This pressure will most likely cause arterial occlusion. C. At this pressure, there is a significant risk of VAP. D. The pressure will cause tracheal necrosis if maintained.

A. The pressure is appropriate for minimal tracheal occlusion.

A sudden decrease in end-tidal CO2 occurs in a mechanically ventilated patient. A repeat analysis yields the same results. Which of the following situations might have accounted for these readings? A. The ventilator circuit has become disconnected. B. There is a leak around the endotracheal tube. C. There is an increase in alveolar dead space. D. The carbon dioxide absorber is exhausted.

A. The ventilator circuit has become disconnected.

A patient in the intensive care unit has the following hemodynamic measurements: CVP (mm Hg) 5 PAP (mm Hg) 29/8 PCWP (mm Hg) 8 BP (mm Hg) 130/70 Cardiac output (L/min) 5.1 Cardiac index (L/min/m2) 2.7 What is the pulse pressure? A. 15 mm Hg B. 21 mm Hg C. 60 mm Hg D. 90 mm Hg

C. 60 mm Hg

The respiratory therapist is in charge of transporting a patient with multiple trauma to a regional trauma center in a fixed wing aircraft. Which of the following should the therapist be most concerned about during the transport? A. Tissue oxygenation B. Pneumothorax C. Pulmonary embolus D. Humidification of the inspired gas

A. Tissue oxygenation

An oxygen-dependent patient uses a nasal cannula at 3 L/min continuously at home. He complains that his liquid oxygen portable device runs out too quickly when he attends church services and prevents him from dining out in restaurants afterwards. Which of the following devices should the respiratory therapist recommend to resolve the issue? A. Use a pulse-dose oxygen delivery system. B. Take a backup E-size oxygen cylinder. C. Reduce the oxygen flow to 2 L/min during church. D. Insertion of a transtracheal oxygen catheter.

A. Use a pulse-dose oxygen delivery system.

A post-operative patient on volume-control ventilation has a chest tube in the left pleural space. While inspecting the chest drainage system, the respiratory therapist notes bubbling in the water seal chamber during the inspiratory phase. The therapist should report this to the physician as A. a persistent bronchopleural fistula. B. a resolved pneumothorax. C. back-pressure from the suction chamber. D. normal function of the water seal chamber.

A. a persistent bronchopleural fistula.

A patient is receiving mechanical ventilation with a PEEP of 10 cm H2O and requires a MRI scan. Before transporting the patient, the respiratory therapist should obtain a A. manual resuscitation bag with a non-ferrous PEEP valve assembly. B. pneumatically powered pressure cycled ventilator with a PEEP valve assembly. C. pneumatically powered demand valve resuscitation device. D. non-disposable resuscitation bag.

A. manual resuscitation bag with a non-ferrous PEEP valve assembly.

All of the following statements are TRUE with regard to cuff inflation techniques EXCEPT A. minimal leak/minimal occlusion volume techniques negate the need for cuff pressure monitoring. B. minimal leak technique allows a small leak at the end of inspiration. C. at minimal occlusion volume, air leakage around the tube cuff should cease. D. cuff pressure should not exceed 35 cmH2O in order to allow circulation to tracheal mucosa.

A. minimal leak/minimal occlusion volume techniques negate the need for cuff pressure monitoring.

A patient reports that he has difficulty breathing while lying in a supine position and prefers to sleep sitting in a chair. The respiratory therapist should record this complaint in the medical record as A. orthopnea. B. platypnea. C. eupnea. D. Kussmaul breathing.

A. orthopnea.

The respiratory therapist is asked to administer 2.5 mg of albuterol to a patient via small volume nebulizer. The medication is available in a 0.5% solution. What volume of albuterol should be administered? A. 0.25 mL B. 0.50 mL C. 1.25 mL D. 2.5 mL

B. 0.50 mL

A patient receiving pressure-controlled ventilation has acute hypoventilation with an ETCO2 of 70 torr. His vital signs include: heart rate 90/min, respiratory rate 18/min, SpO2 94%. Which of the following change(s) will address the situation? 1. Increase the pressure limit 2. Increase the sensitivity 3. Increase the mandatory rate 4. Decrease the inspiratory time A. 1 and 4 only B. 1 and 3 only C. 2 and 3 only D. 1, 2 and 4 only

B. 1 and 3 only

An adult patient is intubated with a 7.0 mm ID endotracheal tube. What size suction catheter should be used to suction this patient? A. 8 French B. 10 French C. 12 French D. 14 French

B. 10 French

Sleep apnea can be defined as repeated episodes of complete cessation of airflow for A. 5 seconds or longer. B. 10 seconds or longer. C. 15 seconds or longer. D. 20 seconds or longer.

B. 10 seconds or longer.

What increase in FEV1 in post-bronchodilator spirometry is needed to confirm reversibility of an obstructive pattern? A. 10% and 100 mL B. 12% and 200 mL C. 15% and 100 mL D. 20% and 200 mL

B. 12% and 200 mL

Thirty minutes after extubation, a patient exhibits moderate stridor. Which of the following should the respiratory therapist recommend? A. Initiate a heated aerosol treatment with saline. B. Administer a racemic epinephrine aerosol treatment. C. Monitor the patient closely for the next hour. D. Immediately reinsert the endotracheal tube.

B. Administer a racemic epinephrine aerosol treatment.

Cardiopulmonary resuscitation has been performed on an adult patient for 15 minutes. The following pattern is seen on the ECG monitor: The respiratory therapist has obtained the following arterial blood gas results: pH 7.02 PaCO2 47 torr PaO2 196 torr HCO3 11 mEq/L SaO2 94% FIO2 1.0 Which of the following should the therapist recommend at this time? A. Increase the minute volume B. Administer sodium bicarbonate C. Decrease the FIO2 D. Administer lidocaine

B. Administer sodium bicarbonate

A patient receiving high frequency oscillation ventilation (HFOV) has demonstrated excess CO2 retention on a recent arterial blood gas. Which of the following could be adjusted to correct this situation? A. Mean airway pressure B. Amplitude (∆P) C. Inspiratory time D. Oxygen concentration

B. Amplitude (∆P)

A post-operative thoracotomy patient is in the PACU and receiving 60% oxygen via a non-rebreather mask. A pulmonary diagnostic assessment reveals the following information: pH 7.43 PaCO2 34 torr PaO2 56 torr HCO3- 22 mEq/L BE -1 SaO2 90% f 25/min. PAP 49/28 mm Hg PWP 5 mm Hg PVR 400 dynes SVR 1150 dynes To improve the patient's pulmonary status, the respiratory therapist should institute A. mechanical ventilation with 100% O2. B. CPAP at 5 cmH2O at 60% O2. C. nitric oxide therapy at 10 ppm. D. heliox therapy at an 80:20 ratio.

B. CPAP at 5 cmH2O at 60% O2.

Which of the following physiologic values would be present in a patient who has proper fluid balance? A. PCWP of 22 mm Hg B. CVP between 3 and 6 mm Hg C. urine output of 20 mL/hr D. increase in body weight

B. CVP between 3 and 6 mm Hg

A 65 year-old female patient with advanced emphysema comes to the ED and is placed on a nasal cannula at 6 L/min. On inspection, the respiratory therapist finds that the patient has become drowsy and less responsive since the oxygen therapy was initiated an hour ago. ABG on 6 L/min are: pH 7.33, PaCO2 64 torr, PaO2 85 torr, HCO3 35 mEq/L. Which of the following should the therapist recommend? A. Leave the patient on the cannula and continue to monitor. B. Change to a 24% Venti-mask and repeat ABG. C. Change to a simple oxygen mask and repeat ABG. D. Prepare the patient for endotracheal intubation.

B. Change to a 24% Venti-mask and repeat ABG.

At the local pulmonary function clinic, a patient is seen for an updated spirometry test for disability qualification as a result of pneumoconiosis. Pulse oximetry is performed with a result of 97%. Arterial blood gases are obtained and the measured SaO2 is 85%. These results are most consistent with A. Restrictive disease B. Cigarette smoking C. Nocturnal asthma D. Obstructive exacerbation

B. Cigarette smoking

A 2-year-old child recently diagnosed with laryngotracheobronchitis requires the administration of 40% oxygen. What is the most appropriate method for delivering the oxygen? A. Oxyhood connected to a blender set at 40% and 10 L/min B. Cool aerosol mask set at 15 L/min and 40% C. Venturi mask set at 40% D. Croup tent at 40%

B. Cool aerosol mask set at 15 L/min and 40%

A healthy adult female can exhale what portion of her forced vital capacity in the first second? A. 50% B. 60% C. 70% D. 80%

C. 70%

A 65 kg spinal cord injured patient has developed atelectasis. His inspiratory capacity is 30% of his predicted value. What bronchial hygiene therapy would be most appropriate initially? A. IS / SMI B. IPPB with normal saline C. postural drainage and percussion D. PEP therapy

B. IPPB with normal saline

A post-operative patient is receiving mechanical ventilation in the ICU at the following settings: VC, A/C; VT 550 mL, respiratory rate 14/min, FIO2 0.50 and 10 cm H2O PEEP. Bedside monitoring results demonstrate that the PvO2 is 35 mm Hg and the SpO2 is 90%. The patient is alert and oriented with stable vital signs. Which of the following should the respiratory therapist recommend? A. Decrease the PEEP. B. Increase the FIO2. C. Initiation diuretic therapy. D. Continue to monitor closely.

B. Increase the FIO2.

A 1600 g neonate is receiving oxygen by oxyhood at an FIO2 of 0.60. The flowmeter is set at 5 L/min. While analyzing the oxygen, the respiratory therapist notices varying FIO2 readings at different locations inside the oxyhood. Which of the following should the therapist do to correct the problem? A. Re-calibrate the oxygen analyzing device B. Increase the flow to the oxyhood C. Place the neonate in an isolette at an FIO2 of 0.60 D. Check the water level of the humidifier

B. Increase the flow to the oxyhood

A 43-year-old female patient has just undergone a total abdominal hysterectomy. The patient arrives in the post anesthesia care unit obtunded with minimal response to painful stimulus. What treatment should the respiratory therapist recommend for this patient? A. Initiate assisted ventilation B. Insert oropharyngeal airway C. Obtain positron emission tomography D. Initiate noninvasive capnography

B. Insert oropharyngeal airway

A patient who is suspected of having a flail chest has been intubated and mechanically ventilated with PEEP therapy. The patient has just received pancuronium bromide. Which of the following ventilator alarms would be most important to set correctly for this patient? A. Peak pressure alarm B. Low PEEP pressure alarm C. I:E ratio alarm D. Low exhaled volume alarm

B. Low PEEP pressure alarm

A trauma patient in the ED is spontaneously breathing oxygen via nasal cannula at 2 L/min. Vital signs are heart rate 110/min, respiratory rate 32/min, blood pressure 90/60 mmHg. The pulse oximeter is reading 88%. Which of the following should the respiratory therapist recommend to maximize the patient's FIO2? A. Simple oxygen mask at 8 L/min B. Non-rebreathing mask at 15 L/min C. CPAP at 10 cmH2O and 0.60 FIO2 D. NPPV of 18/5 cmH2O and 0.60 FIO2

B. Non-rebreathing mask at 15 L/min

The respiratory therapist receives an order to administer a mixture of 80% helium/20% oxygen to a 40 year-old patient in an acute asthmatic episode. Which of the following devices would most effectively deliver the gas mixture to the patient? A. Simple oxygen mask B. Nonrebreathing mask C. Noninvasive ventilator D. Aerosol mask

B. Nonrebreathing mask

A patient receiving mechanical ventilation has developed a temperature of 99.9° F with purulent secretions over the last 12 hours. The respiratory therapist has also noted a steady increase in peak inspiratory pressure. What initial recommendation should be made to address these changes? A. Initiate bronchial hygiene therapy. B. Obtain a sputum gram stain. C. Administer IPV. D. Insert a CASS tube.

B. Obtain a sputum gram stain.

A 19-year-old patient is brought to the emergency department after taking a handful of pills. The patient is obtunded but is making regular, sonorous respiratory efforts. Auscultation reveals coarse rhonchi bilaterally. Which of the following should be done FIRST to assess this patient? A. Obtain a sputum specimen. B. Obtain an ABG. C. Measure peak expiratory flow. D. Determine the Glasgow Coma Score.

B. Obtain an ABG.

A balloon-tipped, flow-directed catheter is positioned in the pulmonary artery with the balloon deflated. Which of the following pressures will be measured by the distal lumen? A. CVP B. PAP C. PCWP D. MAP

B. PAP

A 10-year-old patient with asthma is asked to monitor his airflow improvement following each Beta-2 agonist treatment taken at home. Which of the following should the therapist provide for the patient? A. Turbine pneumotachometer B. Peak flow meter C. Pulse oximeter D. Volume displacement spirometer

B. Peak flow meter

Fine crepitant crackles are most commonly associated with which of the following conditions? A. Bronchitis B. Pulmonary edema C. Pneumonia D. Foreign body aspiration

B. Pulmonary edema

A patient is seen in the Emergency Department for complaints of nausea and vomiting. A nasogastric tube has been inserted and the patient is started on lasix. Which of the following should the respiratory therapist monitor? A. Cardiac enzymes B. Serum electrolytes C. Arterial blood gases D. Cell hydration level

B. Serum electrolytes

Which of the following would be most important to evaluate for a patient who is entering a smoking cessation program? A. Height B. Smoking history C. Weight D. Diet

B. Smoking history

Which of the following would NOT be required to perform a nasal intubation? A. Magill forceps B. Stylet C. Laryngoscope D. Endotracheal tube

B. Stylet

A patient has performed both a forced vital capacity and a slow vital capacity maneuver with the following results: FVC 2.4 L SVC 2.18 L Which of the following statements is TRUE regarding these results? A. The data meets ATS-ERS standards and should be reported. B. The SVC shows poor effort and should be repeated. C. The FVC shows poor effort and should be repeated. D. The patient has obstructive lung disease.

B. The SVC shows poor effort and should be repeated.

An ICU patient's blood pressure is being continuously monitored via an arterial catheter in the left radial artery. The respiratory therapist places the patient in Trendelenburg position for bronchial hygiene therapy and the blood pressure monitor begins to alarm. When the patient is returned to the original position, the blood pressure normalizes. What is the most likely reason for the variation in blood pressure? A. Trendelenburg position causes an elevation in blood pressure. B. The tip of the catheter was below the transducer. C. The catheter needed flushing. D. The catheter was kinked.

B. The tip of the catheter was below the transducer.

What is the correct formula to calculate the static lung compliance of a patient receiving mechanical ventilation? A. Peak pressure ÷ tidal volume B. Tidal volume ÷ (plateau pressure - PEEP) C. Tidal volume ÷ (peak pressure + PEEP) D. (Plateau pressure - PEEP) ÷ tidal volume

B. Tidal volume ÷ (plateau pressure - PEEP)

A 68 year-old patient with advanced emphysema is receiving oxygen by nasal cannula at 1 L/min. The physician has ordered that the patient's SpO2 be maintained at 90%. ABG on 1 L/min are pH 7.34, PaCO2 65 torr, PaO2 55 torr, HCO3 35 mEq/L. What should the respiratory therapist recommend FIRST? A. Initiate NIPPV B. Titrate oxygen flow to the nasal cannula C. Change to a simple mask D. Change to a non-rebreather mask

B. Titrate oxygen flow to the nasal cannula

A heat moisture exchanger is indicated for humidification in which of the following situations? A. Mechanical ventilation in a long-term care facility. B. Transport to a tertiary care center. C. Patient with tenacious secretions. D. Delivery of aerosolized bronchodilators.

B. Transport to a tertiary care center.

The following values are measured on a patient in the outpatient pulmonary function testing center: 1. 2. 3. 4 FVC (L) 3.3 3.4 3.5 3.4 FEV1 (L) 2.3 2.5 2.3 2.4 FEF 25-75% (L/sec) 1.4 1.2 1.3 1.3 Which trial should be recorded as the best test? A. Trial 1 B. Trial 2 C. Trial 3 D. Trial 4

B. Trial 2

A 58 year-old male patient is receiving mechanical ventilation in the ICU on the following settings: VC,SIMV, VT 650 mL, f 12/min, FIO2 0.65, PEEP 10 cmH2O. He has a large amount of thick, yellow secretions. How should the respiratory therapist suction this patient? A. Limit suction time to 5 seconds or less. B. Use a closed-system suction catheter. C. Suction Q1H and PRN. D. Use a 10 Fr suction catheter.

B. Use a closed-system suction catheter.

What is the primary advantage of volume-controlled ventilation as compared to pressure-controlled ventilation? A. VC limits and controls PIP. B. VC provides a constant minute ventilation. C. VC ensures better patient-ventilator synchrony. D. VC delivers a decelerating flow pattern.

B. VC provides a constant minute ventilation.

Which of the following values should the respiratory therapist report as indicative of pulmonary embolism in a patient with acute dyspnea? A. QS/QT of 10% B. VD/VT of 60% C. CL of 60 mL/cm H2O D. RAW of 2.4 cm H2O/L/sec

B. VD/VT of 60%

A 17 year-old patient is receiving 40% oxygen via a Venturi mask following a motor vehicle accident. He suddenly develops acute shortness of breath, is diaphoretic and SpO2 is 85%. Breath sounds are vesicular on the right and extremely diminished on the left. The respiratory therapist should evaluate the patient for the presence of A. atelectasis. B. a pneumothorax. C. arterial hypertension. D. a myocardial infarction.

B. a pneumothorax.

A patient involved in an automobile accident is brought to the ED with tachypnea, tracheal deviation to the right, splinting, asymmetrical chest movement, and decreased breath sounds on the left side. The respiratory therapist should initially A. insert a chest tube. B. administer 100% oxygen via mask. C. perform endotracheal intubation. D. initiate non-invasive positive pressure ventilation.

B. administer 100% oxygen via mask.

Adverse effects of inhaled NO include all of the following EXCEPT A. methemoglobinemia. B. aplastic anemia. C. rebound pulmonary hypertension. D. nitrogen dioxide toxicity.

B. aplastic anemia.

Which of the following is a FALSE statement about self-inflating resuscitation devices? A. A reservoir is utilized to increase the delivered oxygen concentration. B. The therapist can sense changes in the patient's lung compliance and airway resistance. C. A compressed gas source is necessary for the device to operate. D. Excessive gas flow may cause the valve to malfunction.

C. A compressed gas source is necessary for the device to operate.

The respiratory therapist should recommend home apnea monitoring for infants with all of the following situations EXCEPT A. preterm infant with significant apnea periods. B. sibling of a SIDS baby. C. APGAR scores of 4 and 6 at delivery. D. infant with a history of snoring.

C. APGAR scores of 4 and 6 at delivery.

While suctioning an adult patient, the respiratory therapist observes evidence of hypoxemia on the bedside heart monitor. Which of the following are acceptable forms of treatment for the therapist to recommend? A. Perform cardioversion B. Administer lidocaine C. Administer 100% oxygen D. Recommend nitroglycerin

C. Administer 100% oxygen

Noninvasive Positive Pressure Ventilation (NPPV) is contraindicated in the management of which of the following conditions? A. Acute exacerbation of COPD B. Cardiogenic pulmonary edema C. Adult respiratory distress syndrome (ARDS) D. Premature extubation

C. Adult respiratory distress syndrome (ARDS)

While monitoring a newborn utilizing a transcutaneous monitor, you notice a change in PtcO2 from 60 to 142 torr and simultaneously the (PtcCO2) changes from 37 to 2 torr. What is the most likely explanation for these changes? A. Upper airway obstruction B. Poor peripheral perfusion C. Air leak around the sensor D. Device is out of range

C. Air leak around the sensor

A patient is in full cardiopulmonary arrest and after several attempts, the patient is orally intubated with a size 7 mm ID endotracheal tube. The nurse is unable to establish IV access. The ECG monitor shows sinus bradycardia. Which of the following drugs should be administered through the endotracheal tube? A. Lidocaine B. Epinephrine C. Atropine D. Dobutamine

C. Atropine

A 4-year-old girl is admitted to the emergency department with persistent dry coughing. Chest radiograph results reveal normal inspiration with the right lung remaining expanded on expiration. Which of the following should the respiratory therapist recommend at this time? A. Endobronchial intubation B. Arterial blood gas analysis C. Bronchoscopic examination D. Lateral neck radiograph

C. Bronchoscopic examination

The following results are obtained from the pulmonary artery catheter of a patient who collapsed during a visit with a friend in the hospital: CVP 10 cmH2O PAP 33/27 mm Hg PCWP 20 mm Hg BP 108/72 mm Hg Which of the following conditions could be associated with these results? A. Tricuspid valve stenosis B. Right ventricular failure C. Cardiogenic pulmonary edema D. Fluid overload

C. Cardiogenic pulmonary edema

A 32-week gestational age infant is receiving mechanical ventilation for pulmonary interstitial emphysema. The respiratory therapist suspects that a pneumothorax has developed. Which of the following diagnostic procedures should the therapist recommend to confirm this diagnosis? A. Upper airway x-ray B. Capillary blood gas C. Chest radiograph D. Pre- & post-ductal blood gases

C. Chest radiograph

During ventilator rounds in ICU, the respiratory therapist notes that the patient in Room 3 has the following waveform graphic displayed on her ventilator: What action should the therapist take? A. Initiate pressure support. B. Increase respiratory rate. C. Decrease inspiratory time. D. Add an inspiratory plateau.

C. Decrease inspiratory time.

Following administration of oxygen, what additional therapy should the respiratory therapist recommend for a patient with decompression sickness? A. Non-invasive ventilation B. Inhaled nitric oxide C. Hyperbaric oxygen therapy D. Extracorporeal membrane oxygenation

C. Hyperbaric oxygen therapy

A patient who has significant decreases in airflow during sleep but does not have a complete cessation of breathing is having what type of episodes? A. Obstructive sleep apnea B. Dyspnea C. Hypopnea D. Central sleep apnea

C. Hypopnea

A 2000 g neonate is breathing spontaneously on 28% oxygen via an oxyhood with the following arterial blood gas results: pH 7.37 PaCO2 37 torr PaO2 46 torr HCO3- 22 mEq/L Based upon the above information, the respiratory therapist should recommend which of the following? A. Intubate and initiate mechanical ventilation with an FIO2 0.55 B. Intubate and initiate 5 cmH2O CPAP with an FIO2 0.60 C. Increase the FIO2 to 0.35 D. Increase the FIO2 to 0.60

C. Increase the FIO2 to 0.35

A patient is receiving noninvasive positive pressure ventilation by mask. Current settings and arterial blood gas results are as follows: IPAP 10 cm H2O EPAP 5 cm H2O Respiratory rate 12 /min pH 7.31 PaCO2 56 torr PaO2 63 torr HCO3 25 mEq/L BE +1 mEq/L Which of the following should the respiratory therapist recommend? A. Increase the EPAP to 10 cm H2O B. Decrease the EPAP to 3 cm H2O C. Increase the IPAP to 20 cm H2O D. Decrease the IPAP to 5 cm H2O

C. Increase the IPAP to 20 cm H2O

A patient with copious amounts of secretions has required nasotracheal suctioning for the past 36 hours and has now developed mild epistaxis. Which of the following should the respiratory therapist recommend? A. Insert a laryngeal mask airway (LMA) to facilitate suctioning. B. Discontinue nasotracheal suctioning for 24 hours and reassess the patient. C. Insert a nasopharyngeal airway after bleeding has been controlled. D. Insert an oral endotracheal tube to allow for better airway access.

C. Insert a nasopharyngeal airway after bleeding has been controlled.

A patient is admitted to the ED following a motor vehicle accident. On physical exam, the respiratory therapist discovers that breath sounds are absent in the left chest with a hyperresonant percussion note. The trachea is shifted to the right. The patient's heart rate is 45/min, respiratory rate is 30/min, and blood pressure is 60/40 mm Hg. What action should the therapist recommend first? A. Call for a STAT chest x-ray. B. Insert a chest tube into the left chest. C. Needle aspirate the 2nd left intercostal space. D. Activate the medical emergency team to intubate the patient.

C. Needle aspirate the 2nd left intercostal space.

The physician asks the respiratory therapist to set a mechanically ventilated patient's PEEP at an optimal level. The PEEP level is optimal when A. PEEP levels are less than 18 cm H2O. B. PaO2 is 60 torr or greater. C. Oxygen delivery to the tissues is maximal. D. C(a-v)O2 is decreasing.

C. Oxygen delivery to the tissues is maximal.

A balloon-tipped, flow-directed catheter is positioned in the pulmonary artery with the balloon inflated. Which of the following pressures will be measured by the distal lumen? A. CVP B. PAP C. PCWP D. MAP

C. PCWP

A 150 kg (330 lb) patient has been referred to a sleep center for evaluation of sleep apnea and daytime drowsiness. Which of the following is the most appropriate diagnostic procedure? A. Spirometry B. Pulse oximetry C. Polysomnography D. Body plethysmography

C. Polysomnography

A patient with chronic bronchitis is to receive therapy to help remove a large amount of thick purulent secretions. Which of the following would be most helpful? A. Incentive Spirometry B. IPPB C. Postural drainage and percussion D. Small volume nebulizer with normal saline

C. Postural drainage and percussion

All of the following statements regarding an oxygen concentrator are correct EXCEPT A. Provides an unlimited supply of oxygen. B. Increases alveolar oxygen tension. C. Provides 100% oxygen at high flow rates. D. Removes nitrogen from the room air.

C. Provides 100% oxygen at high flow rates.

A sputum sample from an intubated patient with pneumonia has an offensive odor and is described as green and mucopurulent. The respiratory therapist should suspect that this condition is caused by A. a gram positive bacteria. B. respiratory syncytial virus. C. Pseudomonas aeruginosa. D. Pneumocystis jiroveci.

C. Pseudomonas aeruginosa.

A patient has been admitted to the emergency department via ambulance. The cardiac monitor indicates sinus tachycardia. The patient is orally intubated with a size 7.5 mm ID endotracheal tube and is being manually ventilated. What other point of care monitoring should the respiratory therapist recommend for this patient? A. Arterial blood gas analysis B. Transcutaneous monitoring C. Pulse oximetry D. Serum electrolyte analysis

C. Pulse oximetry

During cardioversion, the defibrillator will deliver the synchronized electrical shock on the A. P wave. B. Q wave. C. R wave. D. T wave.

C. R wave.

The respiratory therapist is assisting in the elective intubation of a patient with myasthenia gravis in the ICU. While providing manual ventilation, the self-inflating resuscitation device becomes difficult to compress. Which of the following would the therapist do FIRST? A. Check the inlet valve. B. Check the patient valve. C. Replace the manual resuscitation bag. D. Check for excessive oxygen flow.

C. Replace the manual resuscitation bag.

The following measurements were obtained from a patient with a pulmonary artery catheter in place: CVP 1 mm Hg PAP 10 mm Hg (mean) PCWP 8 mm Hg CI 1.6 L/min/m2 BP 110/90 mm Hg Based on the above information, all of the following values would be decreased EXCEPT A. RVEDP. B. PVR. C. SVR. D. QT.

C. SVR.

The physician has asked the respiratory therapist to monitor the effectiveness of bronchodilator therapy in a patient with asthma. What is the most appropriate parameter to monitor? A. Improvement in level of dyspnea B. Blood gas analysis results C. Serial peak flow measurement D. Improvement in chest x-ray

C. Serial peak flow measurement

A 2100 g neonate is in the NICU and is being monitored with a TcPO2 monitor. The TcPO2 is reading 53 torr with the temperature set at 40oC. The arterial PO2 is 73 torr. Which of the following would best explain the difference in TcPO2 levels? A. There was an error in the arterial blood gas analysis. B. The TcPO2 monitor needs to be repositioned on the neonate. C. The TcPO2 temperature setting is too low. D. The TcPO2 monitor has come off the skin.

C. The TcPO2 temperature setting is too low.

Blood gas results from an apparently healthy patient reveal a pH of 7.37, PaCO2 of 15 torr, PaO2 of 140 torr. The patient is breathing room air and displays no tachycardia, tachypnea, or adventitious breath sounds. Which of the following is an appropriate conclusion to draw on the basis of this information? A. A venous blood sample was drawn. B. The blood gas sample was not cooled prior to analysis. C. The blood gas sample had an air bubble in it. D. The patient's blood gas sample was mixed up with another sample.

C. The blood gas sample had an air bubble in it.

A patient has just been intubated with a naso-tracheal tube and is being manually ventilated. As the respiratory therapist ventilates the patient, he notices that there is no chest movement, minimal breath sounds and air escaping from the mouth as the bag is squeezed. A chest x-ray has determined that the endotracheal tube is in the correct position. What is the most likely cause of this situation? A. The tube is in the esophagus. B. The patient has developed a tracheoesophageal fistula from the intubation process. C. The cuff ruptured during intubation. D. The cuff has herniated over the end of the tube.

C. The cuff ruptured during intubation.

Upon review of the chest radiograph after an elective intubation, the respiratory therapist notes that the distal tip of the endotracheal tube is 3 cm above the carina. How should the therapist interpret this finding? A. The tube is above the recommended position. B. The tube is too long for this patient. C. The tube is in the proper position. D. The tube is below the proper position.

C. The tube is in the proper position.

A patient is suspected of having developed a pulmonary embolus. Which of the following tests should the therapist recommend to confirm the diagnosis? A. (A-a)DO2 B. C(a-v)O2 C. VD/VT D. P50

C. VD/VT

A patient with a size 8.5 mm ID oral endotracheal tube is transported from the Emergency Department to the Intensive Care Unit. The respiratory therapist suspects the tube has changed position during transport. Which of the following is the first step to assess the tube position? A. Chest radiograph B. Bilateral chest auscultation C. Verify symmetrical chest movement D. Diagnostic chest percussion

C. Verify symmetrical chest movement

The respiratory therapist notes a developing hematoma after an arterial blood gas was drawn from the right radial artery. The immediate response is to A. notify the charge nurse. B. apply a pressure dressing. C. apply pressure to the site. D. perform a modified Allen's test.

C. apply pressure to the site.

A 44 year-old patient who suffered a cerebral vascular accident has been moved from Neuro-ICU to the step-down unit. He becomes diaphoretic and his SpO2 suddenly drops from 95% to 88% on a 32% tracheostomy collar. His heart rate is 115/min, respiratory rate is 42/min and his breath sounds are very diminished. The respiratory therapist is unsuccessful in attempting to pass a 12 Fr suction catheter. The therapist should A. increase the suction pressure to 120 mm Hg. B. change to a 10 Fr suction catheter. C. replace the tracheostomy tube. D. orally intubate the patient.

C. replace the tracheostomy tube.

How should the respiratory therapist assemble a Bird Mark 7 respirator to deliver 40% oxygen during an intermittent positive pressure breathing treatment? A. Bleed in oxygen with the air-mix off B. 100% oxygen with the air-mix on C. Air-mix on and attached to a blender set at 40% D. Air-mix off and attached to a blender set at 40%

D. Air-mix off and attached to a blender set at 40%

A 2-year-old child with croup has been intubated for 4 days with a 4 mm ID uncuffed endotracheal tube. Heated aerosol at an FIO2 of 0.30 has been delivered to the patient. The physician asks the respiratory therapist to evaluate the patient for possible extubation. Which of the following would most likely indicate that the patient is ready for extubation? A. The patient is making normal quiet ventilatory efforts. B. A negative sputum culture and sensitivity has been reported. C. The patient's ABG are within normal range. D. Breath sounds are heard around the tube on auscultation.

D. Breath sounds are heard around the tube on auscultation.

A young healthy adult with complaints of intermittent wheezing is seen in the pulmonary clinic. A pre/post bronchodilator spirometry reveals a normal study with no reversibility. Which of the following should the RT recommend? A. Helium dilution study B. DLCO C. Plethysmography D. Bronchial provocation

D. Bronchial provocation

Dynamic hyperinflation is a major concern when using Volume Control, Assist/Control ventilation in patients with which of the following conditions? A. Post-traumatic chest trauma B. Community-acquired pneumonia C. Spinal cord injury D. Chronic bronchitis

D. Chronic bronchitis

Following surgery to correct an abdominal aortic aneurysm, a 54-year-old female patient suddenly develops intense substernal chest pain with severe dyspnea. The pain does not appear to be aggravated by her respirations. Auscultation reveals bilateral, basilar, moist, crepitant rales. The patient appears pale, cold and clammy. Which of the following should the respiratory therapist recommend for initial assessment of this patient? A. Serum electrolytes B. Chest x-ray C. Complete blood cell count D. Electrocardiograph

D. Electrocardiograph

A patient's breathing pattern irregularly increases and decreases and is interspersed with periods of apnea up to 1 minute. Which of the following conditions is the most likely cause of this problem? A. Diabetes insipidus B. Renal failure C. Metabolic acidosis D. Elevated intracranial pressure

D. Elevated intracranial pressure

Which of the following could NOT cause a capnography reading to change from 36 torr to 30 torr? A. Tachypnea B. Hyperventilation C. Pulmonary emboli D. Endotracheal tube positioned in the right mainstem bronchus

D. Endotracheal tube positioned in the right mainstem bronchus

A spontaneous breathing trial was initiated on an intubated, awake, and alert 70 kg (154 lb) patient. After 30 minutes on an FIO2 of 0.30, ABG results are as follows: pH 7.39, PaCO2 44 torr, PaO2 85 torr, and HCO3- 24 mEq/L. The patient's vital signs have remained stable throughout the trial. Which of the following is the most appropriate recommendation? A. Maintain current therapy. B. Initiate NPPV. C. Add 5 cm H2O CPAP. D. Extubate the patient.

D. Extubate the patient.

A 25-year-old, 58 kg (130 lb) female is admitted to ICU after an emergency C-section. The patient is receiving mechanical ventilation via volume control ventilator. The current ventilator settings are: Mode VC, SIMV VT 500 mL f 12 bpm FIO2 0.50 PEEP 5 cm H2O The following laboratory data are available: pH 7.36 PaCO2 44 torr PaO2 50 torr HCO3- 22 mEq/L SaO2 82% CVP 7 cm H2O PAP 13 mm Hg (mean) QT 4.5 L/min. The respiratory therapist should recommend increasing which of the following? A. Tidal volume to 700 mL B. PEEP to 8 cm H2O C. Set rate to 15 bpm D. FIO2 to .60

D. FIO2 to .60

A patient in the intensive care unit has the following hemodynamic measurements: CVP: 12 mm Hg PAP: 48/16 mm Hg PCWP: 15 mm Hg MAP: 99 mm Hg Cardiac Output: 8.0 L/min. Cardiac Index: 4.7 L/min/m2 Which of the following should the respiratory therapist recommend? A. Oxygen B. Dopamine C. Lidocaine D. Furosemide

D. Furosemide

A 52 year-old post-operative patient's chest radiograph demonstrates infiltrates in the posterior basal segments of the lower lobes. Which of the following is the appropriate postural drainage position? A. Head down, patient supine with a pillow under knees B. Patient prone with a pillow under head, bed flat C. Patient supine with a pillow under knees, bed flat D. Head down, patient prone with a pillow under hips

D. Head down, patient prone with a pillow under hips

A fireman is brought to the Emergency Department after being knocked unconscious in a burning office building. What is the best way to improve the oxygenation status of this patient? A. Non-rebreather mask at FIO2 0.75 B. Blood transfusion C. Monitor Co-oximetry every hour D. Hyperbaric oxygen therapy

D. Hyperbaric oxygen therapy

The peak inspiratory pressure on a pressure-cycled ventilator is 30 cm H2O. The respiratory therapist decreases the inspiratory flow. This change would affect the A. rate. B. PEEP. C. expiratory time. D. I:E ratio.

D. I:E ratio.

A 65 year-old patient with end-stage COPD is admitted to the ED with an acute exacerbation. He has a DNI order in his chart. Physical exam reveals that the patient is febrile and has a weak, non-productive cough. Breath sounds reveal bilateral coarse crackles with scattered wheezes. The patient is started on bronchodilator therapy and antibiotics. Serial ABG results are: 7 pm 8 pm FIO2 0.21 0.40 pH 7.30 7.21 PaCO2 70 torr 83 torr PaO2 48 torr 58 torr HCO3 34 mEq/ 34 mEq/L SpO2 78% 89% What should the respiratory therapist recommend? A. Initiate comfort care procedures. B. Decrease FIO2 to 0.35. C. Administer IPV. D. Initiate NPPV.

D. Initiate NPPV.

Which of the following would be the most appropriate therapy for a dyspneic patient who has crepitus with tracheal deviation to the left and absent breath sounds on the right? A. Perform chest physiotherapy B. Administer an IPPB treatment C. Insert an endotracheal tube D. Insert a chest tube

D. Insert a chest tube

A patient is receiving continuous mechanical ventilation through an oral endotracheal tube. The respiratory therapist notes that the high pressure alarm sounds continuously during inspiration and the patient appears to be biting down on the ET tube. Which of the following should the therapist recommend? A. Change to a nasal endotracheal tube B. Add air to the cuff C. Add water to the humidifier D. Insert an oral pharyngeal airway

D. Insert an oral pharyngeal airway

Which of the following statements is TRUE concerning positive expiratory pressure (PEP) therapy? A. It applies expiratory positive airway pressure (EPAP) using a one-way expiratory valve and a one-way inspiratory flow resistor. B. It is used for 5 - 10 minute intervals every hour. C. The inspiratory flow resistor prevents end-inspiratory pressures from rising above zero. D. It may help improve secretion expectoration, decrease hyperinflation and improve airway maintenance

D. It may help improve secretion expectoration, decrease hyperinflation and improve airway maintenance

A mechanically ventilated patient with a tracheostomy tube is on the following settings: PC, SIMV, PIP 30 cmH2O, f 20/min, FIO2 0.60, PEEP 5 cmH2O. The ventilator alarm suddenly begins to sound and on quick examination, the respiratory therapist notices a generalized decrease in breath sounds and a reduction in delivered tidal volume from 650 mL to 500 mL. Which of the following conditions is most likely? A. The patient has been disconnected. B. Complete obstruction of the tracheostomy tube. C. Development of a left-sided pneumothorax. D. Partial obstruction of the tracheostomy tube.

D. Partial obstruction of the tracheostomy tube.

The patient in ICU Bed 6 is noted to have a meniscus in the left chest with a blunted left costophrenic angle on the morning chest radiograph. On physical exam, the respiratory therapist finds that the breath sounds are decreased on the left with a dull percussion note. What treatment should the therapist recommend? A. Insertion of an anterior chest tube. B. Bronchoalveolar lavage. C. Needle aspiration of the 4th left intercostal space. D. Perform a left posterior thoracentesis.

D. Perform a left posterior thoracentesis.

Which of the following measurements is most indicative of pulmonary edema? A. Heart rate of 120/min B. Blood pressure of 92/72 mm Hg C. Pulmonary artery pressure of 25/10 mm Hg D. Pulmonary capillary wedge pressure of 30 mm Hg

D. Pulmonary capillary wedge pressure of 30 mm Hg

A 36 year-old patient is admitted to the ED with a temperature of 38.5° C and suspected pneumonia. The patient has no history of pulmonary disease. Auscultation reveal medium crackles throughout both lungs. Which of the following should be recommended for management of this patient? A. Pre/post bronchodilator study B. Ultrasonic nebulizer treatments C. Manually assisted coughing D. Regular coughing and deep breathing

D. Regular coughing and deep breathing

A pediatric patient on high-flow oxygen therapy is being continuously monitored with a finger pulse oximetry probe. There are frequent and repeated false low SpO2 alarms (less than 90%). Which of the following should the respiratory therapist recommend in this situation? A. Sedate the patient and restrain his arms. B. Reset the low alarm limit to the 80% to 85% range. C. Use a spot-check instead of continuous monitoring. D. Relocate the sensor to the forehead or ear lobe.

D. Relocate the sensor to the forehead or ear lobe.

Which of the following should the respiratory therapist utilize in order to determine the severity of respiratory distress in a newborn? A. Transillumination B. APGAR score C. Ballard score D. Silverman score

D. Silverman score

The respiratory therapist is calibrating a spirometer and checking the volume with a 3.0 liter super syringe. The volumes recorded are: 2.85 L, 2.8 L, and 2.8 L. Based upon the information obtained which of the following is a correct statement? A. Another syringe needs to be used B. Spirometer is accurate C. The plunger was advanced too slowly D. Spirometer may have a leak

D. Spirometer may have a leak

The respiratory therapist receives an order for postural drainage and vibration. With the bed flat, the therapist places the patient in a prone position with pillows under his hips. Which lung segments are being treated with this position? A. Anterior segments of the upper lobes B. Superior segments of the upper lobes C. Posterior basal segments of the lower lobes D. Superior segments of the lower lobes

D. Superior segments of the lower lobes

Indirect calorimetry is performed on a 65-year-old patient to evaluate his nutritional status. It is determined that the patient's RQ is 1.00. This would indicate that the patient's diet consists mostly of A. fats. B. proteins. C. sugars. D. carbohydrates.

D. carbohydrates.

A patient in the ICU is being ventilated with PC, SIMV with a set inspiratory pressure of 62 cm H2O. The respiratory therapist notes that the patient's SpO2 is 92% and PETCO2 is 25 torr. The pressure-volume waveform reveals over-distension. The therapist should A. decrease the respiratory rate. B. decrease the PEEP. C. increase the expiratory time. D. decrease the inspiratory pressure.

D. decrease the inspiratory pressure.

Which of the following devices would produce the greatest humidity output for a patient? A. Bubble humidifier B. Wick-type humidifier C. Large reservoir nebulizer D. Ultrasonic nebulizer

D. Ultrasonic nebulizer

A patient is receiving 60% oxygen at a flow of 8 LPM via a large volume nebulizer and aerosol mask. The patient's inspiratory flowrate is 35 LPM. Which of the following should the respiratory therapist recommend? A. Decrease the flow setting to 5 LPM B. Maintain the current flow setting C. Increase the flow setting to 15 LPM D. Use two nebulizers at a flow of 10 LPM each

D. Use two nebulizers at a flow of 10 LPM each

A patient with a history of asthma presents to the ED in severe respiratory distress and increased accessory muscle use. Vital signs are heart rate 110/min, respiratory rate 32/min and SpO2 of 88% on room air. Bilateral expiratory wheezes are heard on auscultation. The respiratory therapist should recommend initiating A. Xopenex® by MDI B. Pulmacort® by small volume nebulizer C. Salmeterol® by DPI D. Ventolin ® by continuous nebulization

D. Ventolin ® by continuous nebulization

A well-penetrated chest X-ray has which of the following qualities? A. Air bronchograms are prominently displayed. B. Heart borders and pleural spaces are clearly visible. C. Lung parenchyma is black without blood vessels. D. Vertebrae are just visible behind the heart.

D. Vertebrae are just visible behind the heart.

After completing oxygen rounds, the respiratory therapist must clean flowmeters that were removed from rooms where no patients were receiving oxygen therapy. Which of the following methods is most appropriate for disinfection of the flowmeters? A. Steam autoclave B. Soak in acid gluteraldehyde C. Wash in Pasteurmatic D. Wipe with alcohol

D. Wipe with alcohol

The respiratory therapist is working with a patient with COPD in a smoking cessation program. The patient complains of recent weight gain. The therapist should explain that this is not unusual and is a result of A. an increasing feeling of loss of self-control. B. over-reliance on nicotine replacement therapy. C. a need for a prescription for lorazepam. D. a decrease in the patient's metabolism.

D. a decrease in the patient's metabolism.

All of the following could be recommended to treat a patient with severely decreased static lung compliance, EXCEPT A. inverse ratio ventilation. B. appropriate PEEP therapy. C. airway pressure release ventilation. D. aerosolized beta agonist therapy.

D. aerosolized beta agonist therapy.

The sharp rise in exhaled CO2 at the beginning of exhalation on a capnographic tracing is representative of A. tidal volume. B. pulmonary shunt. C. V/Q mismatch. D. alveolar gas that has participated in gas exchange.

D. alveolar gas that has participated in gas exchange.

Twenty-four hours after a patient was intubated, she develops a fever of 99.9°F, a right lower lobe infiltrate, and her white blood cell count is 12,000 per mm3. The respiratory therapist should recommend A. antiviral therapy. B. blood transfusion. C. SABA by small volume nebulizer. D. antibiotic therapy.

D. antibiotic therapy.

Adjusting the inspiratory flow during an IPPB treatment will result in a change in A. respiratory rate. B. peak pressure. C. tidal volume. D. inspiratory time

D. inspiratory time


Conjuntos de estudio relacionados

PHED 1164 Final Exam Study Guide

View Set

Chapter 12 (Close Relationships: Passion, intimacy, and sexuality)

View Set

Chapter 7: Federal Tax Considerations and Retirement Plans

View Set

Chapter 4: Project Integration Management

View Set